You are on page 1of 28

JD-III RULING 1: (1) regular employees or those whose work is

LABOR RELATIONS necessary or desirable to the usual business of


CASE COMPILATION BATCH 1 the employer; they are further classified as:
Security of tenure: Kinds of Employees (1) regular employees by nature of work; and

(2) regular employees by years of service. The former refers


#27 ROWELL INDUSTRIAL to those employees who perform a particular activity which
CORPORATION VS COURT OF APPEALS is necessary or desirable in the usual business or trade of the
G.R. NO. 167714 employer, regardless of their length of service; while the
MARCH 7, 2007 latter refers to those employees who have been performing
the job, regardless of the nature thereof, for at least a year.
CHICO-NAZARIO, J:
(2) project employees or those whose employment has been
FACTS: RIC (petitioner) a tin can manufacturer hired Joel fixed for a specific project or undertaking, the completion or
Taripe (respondent) on November 1999 as a rectangular termination of which has been determined at the time of the
power press machine operator—a position occupied by engagement of the employee or where the work or services
regular employees and the functions of which are necessary to be performed is seasonal in nature and the employment is
to petitioner’s business. Upon taking the job, he was made to for the duration of the season; and
sign a document without any explanation except that it was a
condition for employment. He was not furnished a copy as (3) casual employees or those who are neither regular nor
well. Neither did he receive any benefits under the law and project employees.
the CBA. While Respondent’s case for regularization was
pending, he was summarily dismissed from the job without ISSUE 2: Was respondent a regular employee and thus was
committing any violation, prompting him to file a case for illegally dismissed?
illegal dismissal.
RULING 2: YES. Settled is the rule that the primary
PETITIONER’S CONTENTION: Respondent was a a standard of determining regular employment is the
contractual employee whose services were required due to an reasonable connection between the particular activity
increase in the demand of packaging during Christmas performed by the employee in relation to the casual business
season and to build up stock levels for the early part of the or trade of the employer. The connection can be determined
following year; that on March 2000 his contract expired; that by considering the nature of the work performed and its
the updated information of union members filled up by relation to the scheme of the particular business or trade in
respondent stated that in the last six companies he last its entirety. Respondent held a position which was necessary
worked the reason for leaving was “finished contract” thus and desirable in the usual trade or business of petitioner.
he could not claim ignorance over the document he had Thus, he was a regular employee by the nature of the work
signed upon employment with Petitioner; and that all he performed.
benefits, including SSS were given to him.
It was not proven by petitioner RIC that he was employed
LABOR ARBITER: Dismissed respondent’s complain only for a specific project or undertaking or his
based on a finding that he was a contractual employee whose employment was merely seasonal. Similarly, the position
contract merely expired. and function of power press operator cannot be said to be
merely seasonal. Such position cannot be considered as only
NLRC: Granted respondent’s appeal and declared that his needed for a specific project or undertaking because of the
employment with respondent was regular and his dismissal very nature of the business of petitioner RIC. Indeed,
was illegal. Petitioner’s motion for reconsideration was respondent Taripe is a regular employee of petitioner RIC
denied. and as such, he cannot be dismissed from his employment
unless there is just or authorized cause for his dismissal.
CA: A petition for certiorari under Rule 65 was filed. CA Thus, respondent’s summary dismissal, not being based on
affirmed the decision of the NLRC. The motion for any of the just or authorized causes enumerated under
reconsideration of petitioner was likewise denied. Articles 282, 283, and 284 of the Labor Code, as amended,
is illegal.
ISSUE 1: What are the classifications of an employee under
Article 280 of the Labor Code? ISSUE 3: What are the guidelines that need to be followed in
order for the contract of a fixed term employment not be
violative of security of tenure?
WANGBU2018| BASELERES.CENIZA.CORRO.GAITERA.GARA.MAXINO.VIDAL|Page 1 of 28
RULING 3: Although Article 280 of the Labor Code, as affirming the Labor Secretary’s decision, stating that
amended, does not forbid fixed term employment, it must, “Respondent as a regular employer could not be dismissed
nevertheless, meet any of the following guidelines in order without just cause and expiration of an employment contract
that it cannot be said to circumvent security of tenure: is not one of the just causes provided in the Labor Code.”

(1) that the fixed period of employment was knowingly and ISSUE 1: Are stipulations in employment contracts
voluntarily agreed upon by the parties, without any force, providing for term employment or fixed term employment
duress or improper pressure being brought to bear upon the valid prior the Labor Code? Under the Labor Code?
employee and absent any other circumstances vitiating his
consent; or RULING 1: Prior to the Labor Code: Yes. Term
employment was impliedly but clearly recognized under the
(2) it satisfactorily appears that the employer and employee Termination Pay Law. And prior to this, it was governed by
dealt with each other on more or less equal terms with no the Code of Commerce.
moral dominance whatever being exercised by the former on
the latter. Under the Labor Code: Qualified yes. It is valid when the
period agreed upon knowingly, and voluntarily by the parties
#28 BRENT SCHOOL VS. ZAMORA without force, duress or improper pressure exerted on the
(Fixed Term Employment) employee; and when such stipulations were not designed to
G.R. NO. 48494 circumvent the laws on security of tenure, as the entire
FEBRUARY 5, 1990 purpose behind the law is to prevent such circumvention. If
applied to purposes other than those explicitly stated by its
BERSAMIN, J: framers, the law becomes pointless and arbitrary.

FACTS: Doroteo Alegre (Respondent) was engaged as ISSUE 2: Based on the definition provided by Article 319,
athletic director by Brent School (Petitioner. The contract would a voluntary agreement on a fixed term or period be
fixed a specific term of five years (July 1971-July 1976, valid where the employee “has been engaged to perform
executed before the Labor Code had been promulgated). activities which are usually necessary or desirable in the
Three months before its expiration, Respondent was given a usual trade or business of the employer’?
copy of the report Brent School filed with the DOLE
advising of the termination of his services on the ground of RULING 2: YES. The definition is non sequitur. The
“completion of contract, expiration of definite period of concept of the employee’s duties as being “usually necessary
employment”. or desirable in the usual business or trade of the employer‰
is not synonymous with or identical to employment with a
Upon investigation of Labor Conciliator of the report of fixed term. Logically, the decisive determinant in term
termination, Respondent protested the announced employment should not be the activities that the employee is
termination of his services. Contending that even though his called upon to perform, but the day certain agreed upon by
contract was to terminate on July 1976, his services were the parties for the commencement and termination of their
necessary and desirable in the usual business of his employment relationship, a day certain being understood to
employer. Since his employment had lasted for five years, he be “that which must necessarily come, although it may not
had acquired status of a regular employee and could not be be known when.”
removed without a valid cause. Regional Director considered
Petitioner’s report as an application for clearance to (ART 319: An employment shall be deemed to be without a
terminate (not a report of termination) and accepting the definite period for purposes of this Chapter where the
recommendation of the Labor Conciliator refused to give employee has been engaged to perform activities which are
such clearance and required Respondent’s reinstatement, usually necessary or desirable in the usual business or trade
stating that the ground relied upon by Petitioner was not of the employer, except where the employment has been
sanctioned by PD 442 and a violation of the Circular No. 8 fixed for a specific project or undertaking the completion or
Series of 1989, of the Bureau of Private Schools. termination of which has been determined at the time of the
engagement of the employee or where the work or service to
Brent School filed a motion for reconsideration. The be performed is seasonal in nature and the employment is for
Regional Director denied and forwarded it to the Labor the duration of the season)
Secretary for review. The latter sustained the Regional
Director prompting Petitioner to file an appeal to the office ISSUE 3: Can Respondent be validly dismissed on the
of the President. The office dismissed it for lack of merit, ground that his employment contract had already expired?

WANGBU2018| BASELERES.CENIZA.CORRO.GAITERA.GARA.MAXINO.VIDAL|Page 2 of 28
RULING 3: YES. Respondent Alegre’s employment was are forbidden from agreeing on a period of time for the
terminated upon the expiration of his last contract with Brent performance of such activities.
School on July 16, 1976 without the necessity of any notice.
The advance written advice given the Department of Labor CA: upheld NLRC. Desirability and necessity of the work
with copy to said petitioner was a mere reminder of the done by the petitioners did not make them regular
impending expiration of his contract, not a letter of employees; employment contract for a fixed period is valid
termination, nor an application for clearance to terminate provided it is agreed upon at the same time as the employee's
which needed the approval of the Department of Labor to engagement: INNODATA's operations depends upon job
make the termination of his services effective. In any case, orders from foreign clients, duration of employments could
such clearance should properly have been given, not denied. not permanent but coterminus with projects.

#29 JAMIAS VS. NLRC ISSUE 1: Is stare decisis applicable in this case?
(Employment With A Fixed Term - Manual Editor)
G.R. NO. 159350 RULING 1: NO. The doctrine of stare decisis enjoins
MARCH 9, 2016 adherence to judicial precedents. When a court has laid down
a principle of law as applicable to a certain state of facts, it
BERSAMIN, J.: will adhere to that principle and apply it to all future cases in
which the facts are substantially the same; but when the facts
FACTS: JAMIAS, et. al. were employed by INNODATA are essentially different, stare decisis does not apply because
for a fixed period of one (1) year as expressly stipulated in a perfectly sound principle as applied to one set of facts
the contract. When their contracts expired JAMIAS and the might be entirely inappropriate when a factual variance is
others filed a complaint for illegal dismissal. introduced.

CONTENTION OF PETITIONERS: 1) Innodata made it Servidad and Villanueva case provided for double probation,
appear that they had been hired as project employees in order not a fixed term like this case. The employees concerned, by
to prevent them from becoming regular employees. 2) Lower virtue of a clause incorporated in their contracts, were made
courts erred in not applying stari decisis as regards the nature to remain as probationary employees even if they continue to
of employment settled in Villanueva v. National Labor work beyond the six-month probation period set by law.
Relations Commission (Second Division) and Servidad v. Innodata Philippines, Inc. v. Quejada-Lopez provided for
National Labor Relations Commission. 3) Innodata two periods, 1 year period and another 3 month period that
circumvented the security of tenure protected under Article in reality put the employees under probation.
280 of the Labor Code by providing a fixed term; and that
they were regular employees because the work they In the case at bar petitioners' contracts did not contain similar
performed were necessary and desirable to the business of stipulations, but stipulations to the effect that their
Innodata engagement was for the fixed period of 12 months.

LABOR ARBITER: dismissed the complaint, ruled that ISSUE: Is a fixed period in a contract of employment a
petitioners knowingly signed the contract where the duration circumvention of Article 280 (now 295) of the Labor Code?
of their engagement were clearly stated; fixed term contracts,
being exceptions to Article 280 of the Labor Code, precluded RULING: NO. Article 280 (now 295) contemplates three
their claiming regularization kinds of employees, namely: (a) regular employees; (b)
project employees; and (c) casuals who are neither regular
NLRC: affirmed LABOR ARBITER. Article 280 of the nor project employees. The nature of employment of a
Labor Code did not prohibit employment contracts with worker is determined by the factors provided in Article 280
fixed periods provided the contracts had been voluntarily of the Labor Code, regardless of any stipulation in the
entered into by the parties. The decisive determinant in term contract to the contrary. Article 280 (295) does not preclude
of employment should not be the activities that the employee an agreement providing for a fixed term of employment
is called upon to perform, but the day certain agreed upon by knowingly and voluntarily executed by the parties.
the parties for the commencement and termination of their The test to determine whether a particular employee is
employment relationship, "a day certain" being understood engaged as a project or regular employee is a whether or not
to be "that which must necessarily come, although it may not the employee is assigned to carry out a specific project or
be known when." Where the duties of the employee are undertaking, the duration or scope of which was specified at
activities usually necessary or desirable in the usual business the time of his engagement. There must be a determination
of the employer it does not necessarily follow that the parties of, or a clear agreement on, the completion or termination of
the project at the time the employee is engaged.
WANGBU2018| BASELERES.CENIZA.CORRO.GAITERA.GARA.MAXINO.VIDAL|Page 3 of 28
Otherwise put, the fixed period of employment must be work was not necessary or desirable in respondent’s business
knowingly and voluntarily agreed upon by the parties, which is the manufacture of softdrinks.
without any force, duress or improper pressure being brought ISSUE: Are petitioners regular employees of respondent?
to bear upon the employee and absent any other
circumstances vitiating his consent, or it must satisfactorily RULING: Yes. In determining whether an employment
appear that the employer and employee dealt with each other should be regular or non-regular, the applicable test is the
on more or less equal terms with no moral dominance reasonable connection between the particular activity
whatsoever being exercised by the former on the latter. performed by the employee in relation to the usual business
or trade of the employer. The law itself supplies, "whether
The contracts of the petitioners indicated the one-year the work undertaken is necessary or desirable in the usual
duration of their engagement as well as their respective business or trade of the employer", by looking into the nature
project assignments. There is no indication that petitioners of services rendered and its relation to the general scheme.
were made to sign the contracts against their will, neither did Also for a person engaged in a job for at least one year,
they refute INNODATA's assertion that it did not employ continuous or intermittent, the repeated and continuing need
force, intimidation, or fraudalently induced them to sign. The is deemed by law as indicative of the necessity or desirability
employment of the petitioners who were engaged as project of the activity and hence the employee is regular with respect
employees for a fixed term legally ended upon the expiration to such activity and until its existence. The court deems
of their contract. respondent’s argument as scarcely persuasive. If post
production activities are not necessary and desirable then
#30 PACQUING ET AL VS COCA-COLA only those directly involved in the production of softdrinks
PHILIPPINES maybe held regular employees. This runs against the fact that
there was repeated rehiring of the workers, clearly attesting
(Nature of work to be viewed with the entirety of the trade
to the necessity or desirability of their services and there
or business)
would have been no need to maintain regular truck sales
G.R. NO. 157966
route helpers. The nature of the work performed must be
JANUARY 31, 2008
viewed from a perspective of the business or trade in its
entirety and not on a confined scope. The CA also found each
AUSTRIA-MARTINEZ, J.
of the petitioners to have worked for at least 1 year with
respondent.
FACTS: Petitioners were sales route helpers of respondent
Coca-Cola Bottlers PH employed from various periods
Being regular employees, petitioners are entitled to security
during the 1980s to 1990s. They were a part of a three
of tenure and may only be terminated from employment due
personnel complement comprised of a driver, salesman, and
to just or authorized causes. The failure of respondent to
regular route helper for a delivery truck, their job was to go
show such cause calls for illegal dismissal and thus
with route salesmen on board delivery trucks and undertake
petitioners are entitled to backwages and reinstatement.
the laborious task of loading and unloading softdrink
However the claim for moral damages has no basis, to be
products to various delivery points. Petitioners filed a
awarded the same there must be a showing that the dismissal
complaint against respondent for unfair labor practice and
was attended by bad faith, fraud, oppressive to labor, or
illegal dismissal with claims for regularization and
contrary to morals, good customs or public policy. Without
reinstatement, moral and exemplary damages, and attorney’s
moral damages, there would be no basis for exemplary
fees.
damages and attorney’s fees.
Petitioners alleged that they should be considered regular
employees since the nature of their work was necessary or #31 LU VS. ENOPIA
desirable to respondent’s usual business and directly related G.R. NO. 197899
to respondent’s business and trade and that they have been MARCH 6, 2017
employed for more than one year. On the other hand,
respondent denied liability countering that the petitioners PERALTA, J.
were temporary workers engaged for a five-month period to
FACTS: Respondents were hired from January 20, 1994 to
act as substitutes for an absent regular employee and also that
the work was not necessary or desirable since it merely March 20, 1996 as crew members of a fishing mother boat
involves “post production activities”. owned by petitioner Lu who is the sole proprietor of Mommy
Gina Tuna Resources [MGTR] based in General Santos City.
The Labor Arbiter, NLRC, and CA all ruled against Respondents and Lu had an income-sharing arrangement
petitioners stating that as “cargadores-pahinantes”, their wherein 55% goes to Lu, 45% to the crew members, with an
WANGBU2018| BASELERES.CENIZA.CORRO.GAITERA.GARA.MAXINO.VIDAL|Page 4 of 28
additional 4% as “backing incentive.” They also equally RULING: Yes, respondents were regular employees of the
share the expenses for the maintenance and repair of the petitioner. The fact that petitioner had registered the
mother boat, and for the purchase of nets, ropes and payaos. respondents with SSS is proof that they were indeed his
employees. It was also established that petitioner exercised
Sometime in August 1997, Lu proposed the signing of a Joint control over respondents by assigning a piado and assistant
Venture Fishing Agreement between them, but respondents piado, who every now and then supervise the fishing
refused to sign the same as they opposed the one-year term operations. He also assigned a checker and assistant checker
provided in the agreement. According to respondents, Lu based on the office to monitor and contact every now and
terminated their services right there and then. then the crew at sea through radio. The checker and assistant
checker would advise Lu of the condition and Lu, through
On August 25, 1997, respondents filed their complaint for radio, will then instruct the piado how to conduct the fishing
illegal dismissal, monetary claims and damages. They operations.
alleged that their refusal to sign the Joint Venture Fishing
Agreement is not a just cause for their termination. They also The primary standard for determining regular employment is
asked for a refund of the amount of P8,700,407.70 that was the reasonable connection between the particular activity
taken out of their 50% income share for the repair and performed by the employee in relation to the usual trade or
maintenance of boat as well as the purchase of fishing business of the employer. Respondents’ jobs as fishermen-
materials. crew members were directly related and necessary to
petitioner’s deep-sea fishing business and they had been
On the other hand, Lu denied having dismissed respondents, performing their job for more than one year. /GARA
claiming that their relationship was one of joint venture
where he provided the vessel and other fishing paraphernalia, #32 SAN MIGUEL CORPORATION VS
while respondents, as industrial partners, provided labor by EDUARDO L. TEODOSIO
fishing in the high seas. Lu alleged that there was no
G.R. NO. 163033
employer-employee relationship as it was the piado (master
OCTOBER 2, 2009.
fisherman) who hired petitioners; they were not paid wages
but shares in the catch; and he had no control over the day-
PERALTA, J.
to-day fishing operations. Lu also claimed that respondents
should not be reimbursed for their share in the expenses since
FACTS: Eduardo Teodosio (Eduardo), respondent, was
it was a joint venture.
hired by San Miguel Corporation (SMC), petitioner, as a
casual forklift operator. As an operator, he was tasked with
LABOR ARBITER: LA dismissed the case, finding that
loading and unloading pallet of beer cases within the
there was no employer-employee relationship existing
brewery. He worked from September 1991 to March 1992,
between petitioner and the respondents but a joint venture.
after which he was asked to rest. After a month, he was
rehired for the same position for five to six months, after
NLRC: NLRC affirmed LA’s decision.
which, he was asked to rest again and after three weeks, was
rehired again until August 1993. He was asked to sign an
COURT OF APPEALS: CA reversed NLRC’s decision and
“Employment with a fixed period” contract by SMC with a
found that petitioner exercised control over respondents
term of employment from
based on the following: (1) respondents were the fishermen
crew members of petitioner’s fishing vessel, thus, their
“August 7, 1993 to August 30, 1995, or upon cessation of the
services to the latter were so indispensable and necessary that
instability/fluctuation of the market demand, whichever
without them, petitioner’s deep-sea fishing industry would
comes first.”
not have come to existence much less fruition; (2) he had
control over the entire fishing operations undertaken by the
He continued working without interruption until March 20,
respondents through the piado and assistant piado employed
1995, when he was transferred to the plant’s bottling section
by him; (3) respondents were paid based on a percentage
as a case piler. On April 10, 1995, he sent a letter to SMC
share of the fish catch did not in any way affect their regular
saying that he opposes of his transfer to the bottling section
employment status; and (4) petitioner had already invested
and that he would be more effective as a forklift operator as
millions of pesos in its deep-sea fishing industry, hence, it is
he has already worked there for 3 years and requested for a
highly improbable that he had no control over respondents’
transfer back as a forklift operator but his letter was
fishing operations.
unanswered.
ISSUE: Did an employer-employee relationship exist
between petitioner and respondents?
WANGBU2018| BASELERES.CENIZA.CORRO.GAITERA.GARA.MAXINO.VIDAL|Page 5 of 28
Subsequently, he sent a letter to SMC informing the latter of latter refers to those employees who have been performing
his application to a vacant position in the bottling crew and the job, regardless of the nature thereof, for at least a year. If
his intent of being a regular employee. However, on June 1, the employee has been performing the job for at least one
1995, SMC notified him that his employment shall be year, even if the performance is not continuous or merely
terminated on July 1, 1995 in compliance with their contract. intermittent, the law deems the repeated and continuing need
SMC added this was due to reorganization and streamlining for its performance as sufficient evidence of the necessity, if
of its operation. not indispensability, of that activity to the business.

In another letter, he expressed his dismay and informed SMC Although respondent was initially hired as a casual
that he will not be waiving his right to question his dismissal employee, he attained the status of a regular employee.
and to claim employment benefit as approved in CBA and Respondent was initially hired by SMC on September 5,
company policies. He then signed a Receipt of Release in 1991 until March 1992, rehired for the same position in April
favor of SMC and accepted his separation pay and later on 1992 which lasted for five to six months, after three weeks,
filed a complaint against SMC in NLRC for illegal dismissal rehired again and continued to work as such until August
and under payment. 1993. Thus, he was employed by SMC at least twenty-three
(23) months. The Labor Code provides that a casual
NLRC Bacolod ruled in favor of SMC for lack of merit employee can be considered as a regular employee if said
saying that the contract of employment with a fix period casual employee has rendered at least one year of service
signed by Teodosio was a legitimate exercise of management regardless of the fact that such service may be continuous or
prerogative and also said that since the respondent was not broken.
a union member and not a regular employee of SMC, he was
not entitled to the benefits granted by the existing CBA. Court also added that the nature of respondent’s work is
necessary in the business in which SMC is engaged. SMC is
He appealed in the NLRC Fourth division which dismissed primarily engaged in the manufacture and marketing of beer
the appeal anchoring its decision on the fact that Teodosio products, for which purpose, it specifically maintains a
signed “Receipt and Release” upon receiving his separation brewery in Bacolod City. Teodosio, was engaged as a forklift
pay which was done voluntarily. operator tasked to lift and transfer pallets and pile them from
the bottling section to the piling area. SMC admitted that it
He then filed before the CA which granted his appeal and hired respondent as a forklift operator since the third quarter
reversed the ruling of NLRC. The CA ordered for his of 1991 when, in the absence of fully automated palletizers,
reinstatement and payment of back wages. The CA reasoned manual transfers of beer cases and empties would be
that that the Employment with a Fixed Period contract was extensive within the brewery and its premises. CENIZA
just a scheme of SMC to circumvent respondent’s security of
tenure. The CA concluded that even before the respondent #33 BASAN VS. COCA-COLA BOTTLERS
signed the employment contract, he already attained the PHILIPPINES
status of a regular employee.
(Route helpers perform necessary and desirable functions,
the act of hiring and rehiring for fixed terms short of the
SMC contended that they did not plan on circumventing
probationary period is unjustifiable)
Teodosio’s rights but such was done in good faith and in the
GR NOS. 174365-66
exercise of business judgment, and added that the contract
FEBRUARY 4, 2015
was in accordance with Article 280 of the Labor Code stating
that the employment has been pre-determined, in that the
PERALTA, J.;
duration of the work was contingent upon the cessation of
fluctuating or unstable market demand for beer products,
FACTS: Basan are route helpers of respondent company,
coupled with the automation of brewery operations.
Coca-Cola Bottlers Inc. They filed a complaint for illegal
dismissal alleging that they were dismissed without just
ISSUE: Did Teodosio attain the status of a Regular
cause and prior written notice required by law, to the Labor
Employee?
Arbiter.
RULING: Yes. The court said that regular employees are
Petitiioner, Basan, Assertions: Petitioner, Basan, asserts that
classified into (1)regular employees by nature of work and
they were continuously hired to perform duties necessary and
(2) regular employees by years of service. The former refers
desirable n the usual trade or business and, are therefore
to those employees who perform a particular activity which
regular employees
is necessary or desirable in the usual business or trade of the
employer, regardless of their length of service; while the
WANGBU2018| BASELERES.CENIZA.CORRO.GAITERA.GARA.MAXINO.VIDAL|Page 6 of 28
Respondent, Coca Cola Company, Defenses: Respondent, Furthermore the court elaborates on article 280, (now article
Coca Cola Company disagrees with the contention that 295 of the Labor Code) to wit:
petitioners, as route helpers, were performing functions
necessary or desirable to its business. Respondent, Coca Cola Regular employees are classified into (1) regular employees
Company persistently asserts that petitioner’s assignment by nature of work and (2) regular employees by years of
was temporary in duration and petitioners knew of this. It is service. The Petitioners, in this case, fall under the first kind
respondant company, Coca Cola’s submission that of regular employee.
petitioners, Basan, were only hired as temporary route
helpers to act as substitutes for its absent regular route In rebutting the respondent’s argument based , as well in the
helpers merely for a fixed period . Respondent, Coca Cola case in Magsalin implying that fixed period employment was
Company asserts, that where a fixed period of employment allowed and that the petitioners were employed not as regular
was agreed upon knowingly and voluntarily by the employees but only for a fixed term; The Court said that in
petitioners, the duration of which was made known to them the case of Brent School, Inc v Zamora; a criteria was made
at the time of their engagement, petitioners cannot now claim to prevent circumvention of the employee’s security of
otherwise. tenure to wit:

Respondent, Coca Cola Company presented payslips 1) The fixed period of employment was knowingly and
showing that petitioners merely rendered their services for voluntarily agreed upon by the parties without any force,
periods of less than a year, and thus could not have attained duress, or improper pressure being brought to bear upon the
regulare employment status employee and absent any other circumstances vitiating his
consent
Labor Arbiter decision: Labor arbiter ruled in favor of 2) It satisfactorily appears that the employer and the
petitioners, Basan, upon finding that they were performing employee dealt with each other on more or less equal terms
activities necessary and desirable to the usual business of with no moral dominance exercised by the former or the
respondent coca-cola for more than the period for latter.
regularization, petitioners, Basan are thus considered regular
employees. In the case at bar, the
1.) records of the case is bereft of any proof which show that
NLRC decision: The NLRC affirmed the Labor Arbiter petitioners freely entered into agreements with respondent to
decision and rejected respondent, Cocal Cola’s contention perform services for a specified length of time.
that petitioners, basan were merely employed for a specific 2.) And the court came with the conclusion that ordinary
project or undertaking. NLRC also upheld that the workers such as petitioners herein, are never on equal terms
petitioners, Basan performed work directly connected or with their employees.
necessary and desirable in responde’ts ordinary business of
manufacturing and distributing its softdrink products. The act of respondent Coca-Cola, hiring and rehiring
petitioners for periods short of the legal probationary period
Court of Appeals Decision: The Court of Appeals reversed evidence its intent to thwart security of tenure. It is
the findings of the NLRC and Labor Arbiter. The CA holds unjustifiable to allow respondent to hire and rehire
that performing duties necessary or desirable in the usual petitioners on fixed terms, never attaining regular status.
trade or business is not the only standard for determining the
status of one’s employment. And that fixed term employment #34 GAPAYAO VS FULO
is allowed and enforceable in the jurisdiction. Hence this (FARM WORKER)
petition G.R. NO. 193493
JUNE 13, 2013
ISSUE: Whether or not petitioners are regular employees
and hence entitled to security of tenure SERENO, CJ.:

RULING: Yes. Petitioners, Basan, are regular employees, FACTS: Jaime Fulo was a farm worker in the land of JAIME
the court raises stare decisis citing the case of Magsalin v N. GAPAYAO. Jaime Fulo died of "acute renal failure
National Organization of Working Men. In this case the court secondary to 1st degree burn 70% secondary electrocution"
has categorically declared that the nature of work of route while doing repairs at the residence and business
helpers hired by Coca-Cola Bottlers Philippines Inc. is establishment of petitioner. Petitioner, allegedly due to his
necessary and desirable in its usual business or trade thereby Christian faith, extended financial assistance and entered into
qualifying them as regular employees. a Compromise Agreement with respondent to release him

WANGBU2018| BASELERES.CENIZA.CORRO.GAITERA.GARA.MAXINO.VIDAL|Page 7 of 28
from all liabilities, both civil and criminal, arising from being worked for one (1) season; 2) when they are free to contract
the employer of Fulo. ROSARIO FULO filed a claim for with other farm owners.
social security benefits before SSS. SSS conducted an
investigation and found the deceased to be the employee of For regular employees to be considered as such, the primary
GAPAYAO, SSS ordered the payment but petitioner denied standard used is the reasonable connection between the
employment. FULO elevated the case before the Social particular activity they perform and the usual trade or
Security Commission. business of the employer.

PETITIONER’S CONTENTION: Deceased was not an From year to year, the deceased had been working on
employee, but was rather an independent contractor whose petitioner's land by harvesting abaca and coconut, processing
tasks were not subject to petitioner's control and supervision. copra, and clearing weeds. His employment was continuous
He was an "intermittent worker who was only in the sense that it was done for more than one harvesting
summoned every now and then as the need arose." Deceased season, the tasks were necessary or desirable in the usual
was hired by Adolfo Gamba, a contractor hired by petitioner business of petitioner. The deceased was a construction
to build a building. Allegedly, the Compromise Agreement worker in the building and a helper in the bakery, grocery,
was "extortion camouflaged as an agreement," that hardware, and piggery - all owned by petitioner. This fact
Respondent threatened to refer the matter to the NPA if only proves that even during the off season, the deceased was
Petitioner refused to pay. He maintains he paid the expenses still in the employ of petitioner.
due to his "compassion and sympathy to a distressed person
and his family," and not to admit liability. Pakyaw workers are considered employees for as long as
their employers exercise control over them. It is not essential
RESPONDENT: the "considerable length of time during that the employer actually supervises the performance of
which the deceased was given diverse tasks by petitioner was duties by the employee. It is enough that the former has a
a clear indication of the necessity and right to wield the power. Petitioner as owner of the land
indispensability of her late husband's services to where respondent worked has the right to control his work,
petitioner's business", bolstered by the admission of and this right is also exercised through his farm manager
petitioner himself in the Compromise Agreement that he Amado Gacelo.
was the deceased's employer.
#35 ZENAIDA PAZ VS. NORTHERN
SSC: ruled FULO was an employer working for nine (9) TOBACCO REDRYING CO., INC.
months a year receiving the minimum wage then prevailing.
G.R. NO. 199554
FEBRUARY 18, 2015
CA: Affirmed SSC. It "does not follow that a person who
does not observe normal hours of work cannot be deemed an
CHICO-NAZARIO, J:
employee." Right to control to control is existing (even
exercised) and petitioner entered into a Compromise
FACTS: Northern Tobacco (Respondent) employees, a flue-
Agreement with private respondent and expressly admitted
curing and redrying of tobacco leaves business, employs
therein that he was the employer of the deceased. The CA
approximately 100 employees with seasonal workers tasked
interpreted this admission as a declaration against interest,
to sort, process, store and transport tobacco leaves during the
pursuant to Section 26, Rule 130 of the Rules of Court.
tobacco season of March to September. Zenaida Paz
(petitioner) as a seasonal sorter was paid daily. She was
ISSUE: Is a farm worker who is working under a pakyaw
regularly rehired every tobacco season, signing a seasonal
system considered a regular employee?
job contract at the start of her employment and a pro forma
application prepared by Petitioner in order to qualify for next
RULING: YES. Jurisprudence has identified the three types
season. She was 63 years old when Petitioner was considered
of employees mentioned in Article 280: (1) regular
retired under company policy. A year later she was told she
employees; (2) project or seasonal employees; and (3) casual
would receive her retirement pay. Thus prompting her to file
employees. Farm workers generally fall under the definition
a complaint for illegal dismissal and amending it to include
of seasonal employees. The court have consistently held that
payment of retirement benefits.
seasonal employees may be considered as regular
employees. Regular seasonal employees are those called to
RESPONDENT’S CONTENTION: that no CBA existed
work from time to time. They are in regular employment
between NTRCI and its workers thus computation of
because of the nature of their job, and not because of the
retirement pay is based on Art 287 of the Labor Code; that it
length of time they have worked. Except: 1) when they only
raised the requirement of at least 6 months of service a year
WANGBU2018| BASELERES.CENIZA.CORRO.GAITERA.GARA.MAXINO.VIDAL|Page 8 of 28
for that year to be considered in the retirement pay rehired as a sorter during the tobacco seasons for 29 years
computation; that petitioner had only worked at least six since 1974. These considerations taken together allowed the
months in 1995, 1999 and 2000 out of the 29 years she conclusion that petitioner Paz was a regular seasonal
rendered service. So the computation of her separation pay employee, entitled to security of tenure.
only considered those three years.
#36 DUSIT HOTEL NIKKO VS.
NLRC DECISION: The Labor Arbiter affirmed respondent’s GATBONTON
computation, however the NLRC modified it stating that she
(PROBATIONARY EMPLOYEE-REGULARIZATION)
was to be paid for all the months she was engaged to work
G.R. NO. 161654
for 28 years divided by six (a fraction of six months to be
MAY 5, 2006
considered a year)
QUISUMBING, J.:
COURT OF APPEALS: The CA, although agreeing that
applying the clear text of the law would yield the amount
FACTS: Respondent RENATO M. GATBONTON was
adjudged by the LA, it was a meager amount to support
hired by petitioner DUSIT HOTEL NIKKO to be the Chief
petitioner. Thus the computation for the retirement pay
Steward of its Food and Beverage Department. His contract
decided by the CA was one half month pay multiplied by 29
provided for a three (3) month probationary period. At the
years of service divided by two.
end of his probation, it was extended for another 2 months
for allegedly failing to pass the company standards. But
ISSUE 1: What is the test in determining if an employee is a
before the 2 months extention was completed, at the end of
regular employee or not?
his 4th month, he was terminated. He filed a complaint for
illegal dismissal.
RULING 1: The primary standard is the reasonable
connection between the particular activity performed by the
CONTENTION: PETITIONER contends that Gatbonton
employee in relation to the usual business or trade of the
was still on probation when he was terminated.
employer. The test is whether the former is usually necessary
RESPONDENT contends (1) that the petitioner failed to
or desirable in the usual trade or business of the employer.
establish with substantial evidence that the respondent's
The connection can be determined by considering the nature
probationary employment was extended; (2) that the
of the work performed and its relation to the scheme of the
petitioner failed to establish that the alleged extension was
particular business or trade in its entirety. Also if the
formally communicated to the respondent during his
employee has been performing the job for at least a year,
probationary employment; and (3) that the petitioner failed
even if the performance is not continuous and merely
to establish that the alleged extension was valid and legal.
intermittent, the law deems repeated and continuing need for
its performance as sufficient evidence of the necessity if not
LABOR ARBITER ruled in favor of GATBONTON and
indispensability of that activity to the business. Hence, the
found that he was already regular during his termination, and
employment is considered regular, but only with respect
that there was no evaluation during his purported 3 month
to such activity, and while such activity exists.
extension so no dismissal can be made on the ground that he
did not meet the standards of the company.
Thus, the nature of one’s employment does not depend solely
on the will or word of the employer. Nor on the procedure
NLRC reversed the LA decision and noted that the Personnel
for hiring and the manner of designating the employee, but
Action Form showed respondent's probationary employment
on the nature of the activities to be performed by the
was extended, hence when terminated on the 4th month he
employee, considering the employer’s nature of business and
was still on probation.
the duration and scope of work to be done.
CA reversed NLRC, reinstated LA decision.
ISSUE 2: What kind of employee is Petitioner?
ISSUE: Was respondent validly terminated?
RULING 2: Respondent NTRCI engaged the services of
RULING: NO. Article 281 clearly states, a probationary
petitioner Paz as a seasonal sorter50 and had been regularly
employee can be legally terminated either: (1) for a just
rehiredfrom 1974, until she was informed in 2003 that she
cause; or (2) when the employee fails to qualify as a regular
was being retired under company policy. The services
employee in accordance with the reasonable standards made
petitioner Paz performed as a sorter were necessary and
known to him by the employer at the start of the employment.
indispensable to respondent NTRCI’S business of flue-
Still, the power to dismiss a probationay employee has its
curing and redrying tobacco leaves. She was also regularly
WANGBU2018| BASELERES.CENIZA.CORRO.GAITERA.GARA.MAXINO.VIDAL|Page 9 of 28
limits: First, this power must be exercised in accordance with probationary period. Petitioner filed a complaint for illegal
the specific requirements of the contract. Second, the dismissal due to forced resignation, backwages as well as
dissatisfaction on the part of the employer must be real and damages.
in good faith, not feigned so as to circumvent the contract or
the law; and third, there must be no unlawful discrimination Petitioner contends that at the time of his engagement
in the dismissal. Petitioner did not present proof that the respondent did not make known to him the standards under
respondent was evaluated nor that his probation was validly which he will qualify as a regular employee. On the other
extended. hand, respondent refutes the claim on the basis of a letter
The Personnel Action Plan containing a recommendation for stating that upon effectivity of the probation he and his
extention was prepared beyond his probationary period, and superior are required to jointly define their objectives and on
the recommendation was even termination. There was a the basis of such performance shall be reviewed to assess his
second Personnel Action Plan but it did not contain the competence and work attitude on the 3rd and 5th months of
results of an evaluation, it did not attach a memorandum his probationary employment. That it was petitioner’s failure
containing a recommendation to extend probation, and it did to meet job requirements that cause the termination and
not contain respondent's signature. despite being asked why petitioner simply submitted a
resignation letter.
In the absence of any evaluation or valid extension, the court
cannot conclude that respondent failed to meet the standards The Labor Arbiter and the NLRC both declared petitioner’s
of performance set by the hotel for a chief steward. At the termination as unjustified. But declared petitioner as a
expiration of the three-month period, Gatbonton had become probationary employee, to which respondent is liable only up
a regular employee. A probationary employee engaged to to the unexpired portion of the contract of employment. The
work beyond the probationary period of six months, as CA however ruled that for not having been informed at the
provided under Article 281 of the Labor Code, or for any time of his engagement of the reasonable standards under
length of time set forth by the employer (in this case, three which he will qualify, he was deemed a regular employee but
months), shall be considered a regular employee. Any did not order reinstatement nor award backwages.
circumvention renders the State's policy to protect labor
useless. ISSUE: Was petitioner a regular employee?

#37 ALILING VS FELICIANO RULING: Yes. On the letter-offer, dated June 2, 2004, the
(Probationary employee is deemed regular if not apprised regularization standards or performance norms to be used
by the required standards at the time of his engagement) were still to be agreed upon by petitioner and his supervisor
G.R. NO. 185829 and there was no showing that an agreement has been
APRIL 25, 2012 reached, hence no performance standards to speak of. Not
only that he was assigned to GX sales which is entirely
VELASCO, JR., J. different from the offered Seafreight Sales thus it was not
plausible that the standards relative to his assignment had
FACTS: Petitioner Armando Aliling was offered to be been communicated to him. Under Sec. 6(d) of the IR of
employed as Account Executive (Seafreight Sales) by Book VI Rule VIII-A of the LC, “the employer must make
respondent Wide Wide World Express Corporation known to the employee the standards under which he will
(WWWEC). The offer came w/ a 6 month probationary qualify as a regular employee at the of his engagement.
period with a caveat stating that the performance during such Where no standards are made known to the employee at that
period shall be made as basis for confirmation to regular time, he shall be deemed a regular employee.” Respondent
status. The offer was accepted and training commenced, also allege that the email to petitioner stating his failure to
petitioner was however assigned to handle the sales of a new meet the 80% quota shows the standards for his
product called GX (instead of seafreight), marketing the regularization but the law requires that it must be made
product and finding daily contracts was then his core job. known at the time of his engagement not any time later, the
After a month, the Sales and Marketing Director (San Mateo letter having been sent only more than a month after. Hence,
III) emailed petitioner of his dissatisfying performance (for petitioner was a regular employee at the time of his
not meeting at least 80% of expected sales). Petitioner then engagement.
sent a letter to San Mateo tendering his resignation but then
later on demanded reinstatement and a written apology ISSUE #2: Was petitioner illegally dismissed?
claiming that San Mateo forced him to resign. The HR
Manager (Lariosa) then advised petitioner of the termination RULING: Yes. To justify fully the dismissal of an employee
of his services for non-satisfactory performance during the the employer must as a rule show just cause and that the
employee was afforded due process prior to dismissal.
WANGBU2018| BASELERES.CENIZA.CORRO.GAITERA.GARA.MAXINO.VIDAL|Page 10 of 28
Substantial Due Process. The court held that the alleged PEREZ, J.
failure to meet the 80% sales quota as a mere afterthought on
the part of respondent. It appears that prior to such, FACTS: Petitioner Carvajal was employed as a trainee-teller
management through Lariosa, already informed petitioner of by respondent bank on October 28, 2003 under a six-month
an intent to dismiss him for unauthorized absences and when probationary employment contract. On 10 December 2003,
he was able to satisfactorily explain respondent still the Bank sent petitioner a Memorandum directing her to
proceeded with his plan but this time for “non-satisfactory explain in writing why she should not be subjected to
performance”. Thus, it appears that respondent was at a loss disciplinary action for “chronic tardiness” – petitioner was
to explain the exact just reason for dismissing petitioner. tardy 8 times in November 2003. Petitioner apologized in
writing and explained that she was in the process of making
Even if the same must be justified under Art. 282 of the adjustments regarding her work and house chores. Still, on 6
Labor Code for gross inefficiency, analogous to gross neglect January 2004, a second Memorandum was sent directing the
of duty, as a just cause for dismissal. It must be noted that the petitioner to explain why she should not be suspended for
fixing of the quota must be exercised in good faith for the “chronic tardiness” on 13 occasions for the month of
advancement of respondent’s interests. Respondent fails to December 2003. On 7 January 2004, petitioner submitted her
prove good faith, in fact it appears that the same was a written explanation and manifested her acceptance of the
desperate attempt to lend a semblance of validity to the consequences of her actions. Petitioner was then informed,
illegal dismissal. Not only that, the GX Sales was relatively through a Memorandum, of her suspension for 3 working
new and experimental, petitioner’s performance could not days without pay. Finally, petitioner’s suspension was lifted
then be reasonably quantified. but her employment was terminated effective January 23,
2004. Hence, petitioner’s filing of the Complaint for illegal
Procedural Due Process. Under Sec. 2 Rule XXIII Book V dismissal before the Labor Arbiter. In their position paper,
of the Omnibus Rules Implementing the Labor Code, respondent averred that petitioner was terminated as a
procedural due process requires: a) written notice specifying probationary employee.
grounds for termination with reasonable opportunity to
explain his side; b) hearing or conference where the The Labor Arbiter ruled that petitioner was illegally
employee is given opportunity to respond to the charge; and dismissed as a probationary employee. NLRC affirmed the
c) written notice of termination served on the employee. LA’s Decision with an order for reinstatement. The Court of
Appeals reversed NLRC’s ruling and held that petitioner was
Here, the first and second notice requirements have not been rightfully dismissed for failure to meet employment
properly served, respondent has this adverted memo on Sept. standards.
20, 2004 supposedly informing petitioner of the likelihood of
his termination but no proof was presented that such letter ISSUE 1: Did petitioner become a regular employee?
was sent to and received by petitioner. In fact petitioner went
as far as to tag such memo as fabrication. The record was RULING: No. A probationary employee, like a regular
devoid of any showing of a hearing or conference having employee, enjoys security of tenure. However, in cases of
been conducted. probationary employment, aside from just or authorized
causes of termination, an additional ground is provided under
Backwages, separation pay, damages Article 281 of the Labor Code, i.e., the probationary
employee may also be terminated for failure to qualify as a
Having been found a regular employee, petitioner is entitled regular employee in accordance with reasonable standards
to backwages from the time of illegal dismissal until the made known by the employer to the employee at the time of
finality of the decision. Additionally separation pay is reward the engagement. Thus, the services of an employee who has
in lieu of reinstatement on the ground of strained been engaged on probationary basis may be terminated for
relationship. But only attorney’s fees were awarded for any of the following: (1) a just or (2) an authorized cause and
having been forced to litigate. Moral and exemplary damages (3) when he fails to qualify as a regular employee in
were not awarded for failure to show bad faith (?). accordance with reasonable standards prescribed by the
/DIORSON employer.

#38 CARVAJAL VS. LUZON It is evident that the primary cause of petitioner’s dismissal
DEVELOPMENT BANK from her probationary employment was her “chronic
tardiness.” Punctuality is a reasonable standard imposed on
G.R. NO. 186169
every employee, whether in government or private sector. As
AUGUST 1, 2012
a matter of fact, habitual tardiness is a serious offense that
may very well constitute gross or habitual neglect of duty, a
WANGBU2018| BASELERES.CENIZA.CORRO.GAITERA.GARA.MAXINO.VIDAL|Page 11 of 28
just cause to dismiss a regular employee. Assuming that On said date, Gala and his workmates were instructed to
petitioner was not apprised of the standards concomitant to replace a worn out electric pole at the Pacheco Subdivision
her job, it is but common sense that she must abide by the in Valenzuela City. Gala was asked to join the Truck No.
work hours imposed by the bank. As the Court has aptly 1891 under Nemecio Hipolito. When they arrived at the work
stated in Aberdeen Court, Inc. v. Agustin, Jr., the rule on site, the Truck No 19837 supervised by Zuniga was already
reasonable standards made known to the employee prior to there and its linemen were already at work. Gala as well as
engagement should not be used to exculpate a probationary the other crew of Truck No. 1891 were instructed to help in
employee who acts in a manner contrary to basic knowledge in digging a hole for the pole.
and common sense, in regard to which there is no need to
spell out a policy or standard to be met. While at work Norberto Llanes (Norberto), not an employee
of MERALCO, arrived. He was known to the foremen as he
ISSUE 2: Was the petitioner’s dismissal done without due was seen to be conversing with him. Norberto boarded the
process? truck without being stopped and took out some electrical
supplies. Aside from Gala, the foremen and the other linemen
RULING: No. As elucidated in Philippine Daily Inquirer, who were at the worksite when the incident happened were
Inc. v. Magtibay, Jr.: Unlike under the first ground for the later charged with misconduct and dishonesty for their
valid termination of probationary employment which is for involvement in the incident.
just cause, the second ground [failure to qualify in
accordance with the standards prescribed by employer] does A MERALCO surveillance taskforce was formed to monitor
not require notice and hearing. Due process of law for this their activities and have it recorded in camera. The task force
second ground consists of making the reasonable standards was composed of Joseph Aguilar, Ariel Dola and Frederick
expected of the employee during his probationary period Riano. MERALCO called for am investigation but Gala
known to him at the time of his probationary employment. denied involvement saying that if his supervisors were
By the very nature of a probationary employment, the committing the wrongdoing, he was not involved. He
employee knows from the very start that he will be under claimed that: (1) he was at some distance away from the
close observation and his performance of his assigned duties trucks when the pilferage happened; (2) he did not have an
and functions would be under continuous scrutiny by his inkling that an illegal activity was taking place since his
superiors. It is in apprising him of the standards against supervisors were conversing with Llanes, giving him the
which his performance shall be continuously assessed where impression that they knew him; (3) he did not call the
due process regarding the second ground lies, and not in attention of his superiors because he was not in a position to
notice and hearing as in the case of the first ground. do so as he was a mere lineman; and (4) he was just following
instructions in connection with his work and had no control
In the case at hand, respondent complied with the basic in the disposition of company supplies and materials. He
requirements of due process by reminding petitioner, through maintained that his mere presence at the scene of the incident
memoranda, of the standards that should be observed in was not sufficient to hold him liable as a conspirator. Despite
aspiring for regularization. /GARA this, he was still terminated. Gala then filed for Illegal
Dismissal to the Labor Arbiter.
#39 MANILA ELECTRIC COMPANY VS.
JAN CARLO GALA The Labor Arbiter dismissed the complaint for lack of merit
ruling that Gala’s participation in the pilferage of Meralco’s
G.R. NOS. 191288 & 191304
property rendered him unqualified to become a regular
FEBRUARY 29, 2012
employee.
BRION, J.:
Gala appealed to the NLRC who reversed the ruling of the
labor arbiter stating that Gala had been illegally dismissed,
FACTS: Respondent, Jan Carlo Gala (Jan) started his
since there was no concrete showing of complicity with the
employment with Petitioner Manila Electric Company
alleged misconduct/dishonesty. However, it ruled out the
(MERALCO) on March 2, 2006 as a PROBATIONARY
reinstatement, stating that his tenure lasted only up to the end
LINEMAN. He first served as a member of Truck No. 1823
of his probationary period but it awarded him backwages and
supervised by Foreman Narciso Matis. After one month, he
attorney’s fees.
joined the crew of Truck No. 1837 under the supervision of
Foreman Raymundo Zuñiga, Sr. On July 27, 2006, Gala was
Gala moved for partial reconsideration stating that he should
dismissed for alleged complicity in pilferages of
be reinstated as well as paid for damages and interest, while
MERALCO’s electrical supplies specifically in the incident
MERALCO for pointing out the error in finding Gala
on May 25, 2006.
illegally dismissed. NLRC denied both motions.
WANGBU2018| BASELERES.CENIZA.CORRO.GAITERA.GARA.MAXINO.VIDAL|Page 12 of 28
appointment up to the same calendar date of the 6th moth
Both parties filed for a review under Rule 65 before the CA following.)
to which it ruled denying MERALCO’S petition while G.R. NO. 205278
partially granting Gala. CA stated that he was illegally JUNE 11, 2014
dismissed and the same was supported by evidence while
also stating that there were no sufficient evidence showing MENDOZA J;
Gala’s complicity in the alleged pilferage. Subsequently, it
modified NLRC’s decision and ordered for Gala’s FACTS: Respondent, Juvenstein Mahilum, was the Vice
reinstatement. President for Sales and Marketing for Petitioner, Philippine
Spring Water Resources’s (PSWRI)’s Bulacan plant.
ISSUE: Is Gala a regular employee entitled to reinstatement?
Petitioner, PSWRI manufactures, sells and distributes bottled
RULING: No. He failed to qualify as a regular employee. mineral water.
There is substantial evidence supporting Meralco’s position
that Gala had become unfit to continue his employment with In 2004 Respondent, Mahilum, was designated as over-all
the company. Gala was found, after an administrative chairman for the company’s Inauguration and Christmas
investigation, to have failed to meet the standards expected party of its Bulacan plant. On the day of the program
of him to become a regular employee and this failure was Mahilum ‘s attention was called when Danilo Lua, the
mainly due to his “undeniable knowledge, if not President and Chief Executive Officer of Bulacan, was
participation, in the pilferage activities done by their group, furious because he was not recognized during the program.
all to the prejudice of the Company’s interests.”
Mahilum was required to explain why Lua was not
As a probationary employee, his overall job performance and recognized the next day. He was then placed under
his behavior were being monitored and measured in preventive suspension for 30 days. He then received a copy
accordance with the standards laid down in his probationary of a Memorandum terminating his services. He was made to
employment agreement. Under paragraph 8 of the execute a release, waiver and quitclaim in favour of the
agreement, he was subject to strict compliance with, and non- company and Lua. Mahilum filed a complaint for illegal
violation of the Company Code on Employee Discipline, dismissal with prayer for reinstatement, payment of
Safety Code, rules and regulations and existing policies. Par. backwages and damages.
10 required him to observe at all times the highest degree of
transparency, selflessness and integrity in the performance of Assertions and Defenses:
his duties and responsibilities, free from any form of conflict
or contradicting with his own personal interest. Respondent, Mahilum argues that he was illegally suspended
and thereafter constructively dismissed. And he was forced
On the whole, the totality of the circumstances obtaining in to sign the waiver. He argued that he was a regular employee
the case shows that Gala could not but have knowledge of and was entitled to security of tenure, having been hired on
the pilferage of company electrical supplies. He was June 2004 and dismissed on February 1, 2005, thereby
complicit in its commission, if not by direct participation, already serving the company for 8 months at the time of his
certainly, by his inaction while it was being perpetrated and dismissal
by not reporting the incident to company authorities. Thus,
there is substantial evidence to Petitioner, PSWRI argues that Mahilum was not a regular
support the conclusion that Gala does not deserve to remain employee at the time of his dismissal because his
in Meralco’s employ as a regular employee because he probationary status would end only if he could satisfactorily
violated his probationary employment agreement, especially perform his duties and functions defined in their Personnel’s
the requirement for him “to observe at all times the highest Manual/Company House Rules of Discipline. And this
degree of transparency, selflessness and integrity in the suspensive condition failed to arise.
performance of their duties and responsibilities” CENIZA
Decisions:
#40 PHILIPPINE SPRING WATER Labor Arbiter
RESOURCES, INC. VS. COURT OF Reespondent Mahilum filed with.Labor Arbiter and Labor
Arbiter Dismissed, respondent, Mahilum’s complaint on the
APPEALS ground that the quitclaim he had exercised bared his right to
(Probationary employees also enjoy security of tenure, The question his dismissal under the principle of estoppel.
date counting towards regularization starts from date of
NLRC:
WANGBU2018| BASELERES.CENIZA.CORRO.GAITERA.GARA.MAXINO.VIDAL|Page 13 of 28
Respondent Mahilim appealed the decision to the NLRC and The court cannot likewise subscribe to the idea that Mahilum
NLRC ruled in favor of respondent, Mahilum’s appeal, on failed to qualify as a regular employee when he failed to
the ground that the quit claim did not bar the institution of perform at par with the standards made know by the
the case for illegal dismissal. The Labor Arbiter’s company to him. It is clear that the primary cause of
consideration of the waiver did not constitute a reasonable Mahilum’s dismissal was borne out of his alleged lapses as
settlement of his cause of action. The amount he received chairman for the inauguration of the Bulacan plant
from the quitclaim were benefits he already earned by virtue company’s Christmas party.
of his employment and not in consideration of his separation
from service. In line with this, the NLRC ruled that Being a regular employee he can only be dismissed from
respondent, mahilum was illegally dismissed. While service for causes provided for in Art. 282 (now Art. 297) of
Mahilum may have failed to discharge his duties as chairman the Labor Code. His designation as the chairman of the whole
of the inauguration of the Bulacan plant, the same was not affair did not form part of his duty as a supervisor. And the
sufficient to deprive him of his employment on the ground of court found that his failure to effectively discharge the party
loss of confidence. was due to mere inadvertence and the mistaken belief that he
had properly delegated the details of the program to another
Court of Appeals: officer.
Petitioner, PSWRI filed a petition of certitorari with the CA
and CA ruled in favour of PSWRI. The termination was
illegal but by signing the quit claim, Respondent Mahilumm #41 UNIVERSIDAD DE STA. ISABEL VS
unwittingly discharged the company of its liability. SAMBAJON, JR.
(Fixed term must give way to probationary status if contract
Court of Appeals, Reconsideration:
not specifically used for the fixed-term it offers)
Respondent, Mahilum filed fore reconsideration and CA, in
G.R. NO. 196280 & 196286
reconsideration, ruled in favour of Mahilum. Finding that ,
APRIL 2, 2014
the quitclaim was void for having no consideration at all and
that he was illegally dismissed.
VILLARAMA, JR., J.
Hence,petioiners, PSWRI and Lua filed a petition for
FACTS: Petitioner is a nonstock nonprofit religious
certiorari to the SC.
education institution which hired respondent Marvin
Sambahon, Jr. as a full-time college faculty member with the
ISSUE: Whether or not Mahilum is a regular employee and
rank of Assistant Professor on probationary status from Nov.
was illegally dismissed
1, 2002 to March 30, 2003. Upon expiry, petitioner continued
giving teaching loads to respondent remaining a full-time
RULING: Yes, he was a regular employee and was illegally
faculty member of the Dept. of Religious Education up to
dismissed.. The court Used Art. 281 of the Labor Code (Now
March 31, 2005. Sometime in 2003, respondent submitted a
Article 296 of the Labor Code).Which states that,
special order from the CHED along with his credentials for
probationary employment shall not exceed 6 months from
his master’s degree for salary adjustment. As a result, his
the date the employee started working…. An employee who
salary was increased starting October 2004 and was re-
is allowed to work after a probationary period shall be
ranked to Associate Professor. However, respondent
considered a regular employee.
demands that the increase be made effective as of June 2003
and demanded payment for salary differential. Petitioner
A probationary employee like a regular employee enjoys
denies the same stating that re-ranking is done only every
security of tenure and may be terminated for (1) a just or (2)
two years and that they are under no obligation to re-rank
authorized cause and (3) when he fails to qualify as a regular
him or adjust his salary with the special consideration only
employee in accordance with reasonable standards
done on October 2004 and no way can it be retroactive. Upon
prescribed by the employer.
respondents insistence, a dialogue was held to resolve the
conflict. Later on, Feb. 2005 respondent received a letter of
Mahilum was a regular employee. He served the company
termination stating that his probationary appointment will
for 8 months. Having been allowed to work after the lapse of
not be renewed when it expires at the end of March, 2005.
the probationary period, Mahilum became a regular
Respondent files a complaint for illegal dismissal.
employee. Being a regular employee he was entitled to
security of tenure. The computation of the 6 month
The Labor Arbiter, NLRC, and CA ruled in favor of
probationary period is reckoned from the date of
respondent stating that there was no just or authorized cause
appointement up to the same calendar date of the 6th month
in the termination of respondent’s probationary employment.
following.
WANGBU2018| BASELERES.CENIZA.CORRO.GAITERA.GARA.MAXINO.VIDAL|Page 14 of 28
The NLRC and CA went even further stating that respondent Hence, when probationary status overlaps with a fixed-term
has acquired permanent status for having been hired on a contract not specifically used for the fixed-term it offers, Art.
probationary status from Nov. 1, 2002 to March 30, 2003 and 281 assumes primacy and the fixed-period character must
his employment continued beyond said period and lasted for give way.
5 consecutive semesters.
ISSUE: Was respondent illegally dismissed?
ISSUE: Was respondent a probationary or fix-termed
employee? RULING: Yes. Despite being a probationary employee they
are still entitled to the constitutional protection of security of
RULING: Respondent was a probationary employee. A tenure during and before end of probation that they may be
probationary employee is one who is on trial by the employer terminated only for: a) just cause; b) authorized cause; and c)
during which the employer determines whether or not said failure to qualify as a regular employee in accordance w/
employee is qualified for permanent employment. The word reasonable standards prescribed by the employer;
probationary as used to describe the period of employment
implies the purpose of the term or period, but not its length. Thus, while no vested right to a permanent appointment
It is within the exercise of the right to select his employees accrues in favor of respondent he nevertheless acquires a
that the employer may set or fix a probationary period within limited tenure. During said period, he cannot be terminated
which the latter may test and observe the conduct of the except for those mentioned above. And in this case, no such
former before hiring him permanently. The law, however, cause was given by petitioner if anything they simply
regulates the exercise of this prerogative to fix the period of terminated respondent after five semesters of satisfactory
probationary employment, a maximum of 6 months for service. /DIORSON
ordinary employees and 3 years in the academe.
#42 CANADIAN OPPORTUNITIES
In the case of Magis Young Achievers Learning Center the UNLIMITED INC VS. BART Q. DALANGIN
court explained that it is common practice for the employer
G.R. NO. 172223
and teacher to enter into a contract effective for 1 school year
FEBRUARY 6, 2012
and then renew the same so that at the end of the third school
year the employer may decide whether to extend a permanent
BRION, J:
appointment to the employee or not. For this entire 3 year
period the teacher remains under probation and the school
FACTS: Bart Dalangin, respondent filed a complaint for
cannot be compelled to renew his employment contract upon
illegal dismissal against Canadian Opportunities Unlimited,
expiry. However, petitioner argues that respondent’s
Inc., petitioner. Petitioner provides assistance and related
probationary period expired after each semester and hence it
services to applicants for permanent residence in Canada.
was not obligation to renew his services at the end of the 5th
Respondent was hired as Immigration and Legal Manager.
semester. To the court, petitioner is treating the contracts as
He was placed on probation for six months. His tasks
fixed-term employment contracts. This is not the case since
involved principally the review of clients’ applications for
under Magis the probationary period is for the entire duration
immigration to Canada to ensure that they are in accordance
of the 3 year period.
with Canadian and Philippine laws. Less than a month into
his employment, respondent was terminated through a
But in the case of Mercado vs AMA Computer College, the
memorandum, declaring him unfit and unqualified to
court held that the school may determine for itself to use
continue as Immigration and Legal Manager.
fixed-term employment contracts as its medium for hiring its
teachers. Nevertheless, the teacher’s probationary status
During the compulsory arbitration proceeding, respondent
cannot be disregarded simply because their contracts were
contends that due to his failure to attend a seminar that was
fixed-term. In a sense probationary and fixed-term are
scheduled on Saturday beyond the prescribed work hours he
somewhat the same in a sense that both refers to periods but
was required to explain his absence. And that when he met
the similarity ends there since being on probation connotes a
with the managing director regarding the matter, he was told
process of testing and observing the character/abilities of
that his service was terminated because they cannot keep
person new to the job. It appears to the court, that the fixed-
“people who are hard headed and who refuse to follow orders
term contract was conveniently used by the school to define
from management”. Petitioner, contends that respondent as
and regulate its relations with its teachers during their
probationary employee could be terminated should he fail the
probationary period. This reconciliation has to be made so
standards to qualify him as a regular employee. It points out
that schools may not wreck the scheme that the Constitution
that respondent lack enthusiasm towards his work, was
and the Labor Code has established.

WANGBU2018| BASELERES.CENIZA.CORRO.GAITERA.GARA.MAXINO.VIDAL|Page 15 of 28
indifferent to his co-workers and refused to follow company Dalangin overlooks the fact, wittingly or unwittingly, that he
procedures. offered glimpses of his own behavior and actuations during
his four-week stay with the company; he betrayed his
LABOR ARBITER: Ruled that the dismissal was illegal. The negative attitude and regard for the company, his co-
charges against respondent were not established by clear and employees and his work.
substantial proof.
• Refusal to attend the company’s “Value’s Formation
NLRC: Reversed the LA’s decision as the dismissal was a Seminar” after learning that it had no relation to his job
valid exercise of the company’s management prerogative. which highlights his lack of interest in familiarizing himself
with the company’s objectives and policies.
CA: The NLRC erred when it ruled that the dismissal was • When his attendance was insisted to encourage his
not illegal, finding that petitioner was not able to prove their co-employees, he pointed out the marked differences
claim with substantial evidence. between their jobs.
• Negative working habits: prolonged lunch breaks
ISSUE 1: Who is a probationary employee? and going out of the office without leave.
• Incompetence in handling a client’s application.
RULING 1: A probationary employee, as understood under
Article 281 of the Labor Code, is one who is on trial by an As we stressed earlier, we are convinced that the company
employer, during which, the latter determines whether or not had seen enough from Dalangin’s
he is qualified for permanent employment. A probationary actuations, behavior and deportment during a four-week
appointment gives the employer an opportunity to observe period to realize that Dalangin would be a liability rather than
the fitness of a probationer while at work, and to ascertain an asset to its operations.
whether he would be a proper and efficient employee.
We, therefore, disagree with the CA that the company could
The essence of a probationary period of employment not have fully determined Dalangin’s performance barely
fundamentally lies in the purpose or objective of both the one month into his employment. As we said in International
employer and the employee during the period. While the Catholic Migration Commission, the probationary term or
employer observes the fitness, propriety and efficiency of a period denotes its purpose but not its length. To our mind,
probationer to ascertain whether he is qualified for four weeks was enough for thecompany to assess Dalangin’s
permanent employment, the latter seeks to prove to the fitness for the job and he was found wanting. In separating
former that he has the qualifications to meet the Dalangin from the service before the situation got worse, we
reasonable standards for permanent employment. The “trial find the company not liable for illegal dismissal.
period” or the length of time the probationary employee
remains on probation depends on the parties’ agreement, but #44 MAGIS YOUNG ACHIEVERS
it shall not exceed six (6) months under Article 281 of the LEARNING CENTER VS MANALO
Labor Code, unless it is covered by an apprenticeship
(Probationary employees are entitled to security of tenure)
agreement stipulating a longer period. The services of an
G.R. NO. 178835
employeewho has been engaged on a probationary basis may
FEBRUARY 13, 2009
be terminated for a just cause or when he fails to qualify as a
regular employee in accordance with reasonable standards
NACHURA, J.
made known by the employer to the employee at the time of
his engagement. An employee who is allowed to work after
FACTS: Respondent Adelaida Manalo was hired as teacher
a probationary period shall be considered a regular
and acting principal of petitioner on April 18, 2002. It
employee.
appears that on March 29, 2003 she wrote a letter of
resignation addressed to the directress of petitioner stating
ISSUE 2: Was respondent validly dismissed?
the cause thereof as personal and family reasons. On March
31, 2003 she received a letter of termination from petitioner
RULING 2: Yes. Contrary to the CA’s conclusions, we find
stating that due to a cost-cutting scheme a systematic
substantial evidence indicating that the company was
reorganization of the school was in order and the position of
justified in terminating Dalangin’s employment, however
principal was to be abolished the following school year and
brief it had
hence they can no longer renew respondent’s contract.
been. Time and again, we have emphasized that substantial
Respondent then instituted a complaint against petitioner for
evidence is such relevant evidence as a reasonable mind
illegal dismissal.
might accept as adequate to support a conclusion.

WANGBU2018| BASELERES.CENIZA.CORRO.GAITERA.GARA.MAXINO.VIDAL|Page 16 of 28
Respondent contends that her termination violated the
provisions of her employment contract and that the alleged As for the actual probationary period, the parties presented 2
abolition of the position of principal was not among the copies of the agreement one by the petitioner showing a
grounds for termination under Art. 282 LC. On the other period of 1 year and one by the respondent with the duration
hand, petitioner countered that the one-year probationary left blank. Due to the dubious situation the court applied Art.
period from April 1, 2002 to March 3, 2003 had already 1702 which construes labor contracts in favor to labor and
lapsed and she failed to meet the criteria set by the school. upheld the respondents copy as the more accurate copy
especially since there was a disparity in the handwriting of
The labor arbiter dismissed the complaint stating that on the petitioner’s copy. Thus, the law deems the probationary
contrary she resigned and thus it was hard for them to period as 3 years.
imagine complainant accede to sign a resignation letter as a
precondition to her hiring considering her educational Finally, despite being probationary respondent was
background. The NLRC and CA reversed the arbiter’s nevertheless illegally dismissed. Probationary employees
judgment ordering the reinstatement of respondent to be enjoy security of tenure during the term of their probationary
credited with 1 year service of probationary employment. employment such that they may only be terminated for cause
as provided for by the law or if at the end of the period fails
ISSUE: Was respondent illegally dismissed? to meet reasonable standards set at the time of engagement.
Thus the abolishment of principal was not a valid cause for
RULING: Yes. A probationary employee or probationer is termination, nowhere in the letter was respondent informed
one who is on trial for an employer, during which the latter that her performance was not satisfactory. /DIORSON
determines whether or not he is qualified for permanent
employment. The probationary employment is intended to #45 MERCADO VS. AMA COMPUTER
afford the employer an opportunity to observe the fitness of COLLEGE-PARAÑAQUE CITY, INC.
a probationary employee while at work, and to ascertain
G.R. NO. 183572
whether he will become an efficient and productive
APRIL 13, 2010
employee. The law regulates the exercise of the employer’s
prerogative to fix the period of probationary employment.
BRION, J.
While there is no statutory cap on the minimum term of
probation, the law sets a maximum “trial period”. As a
FACTS: The petitioners were faculty members who started
general rule, Art. 281 provides that probationary
teaching at AMACC on May 25, 1998. The petitioner
employment shall not exceed six (6) months from the date
Mercado was engaged as a Professor 3, while petitioner
the employee started working but under Sec. 92 of the 1992
Tonog was engaged as an Assistant Professor 2. On the other
Manual of Regulations for Private Schools the period shall
hand, petitioners De Leon, Lachica and Alba, Jr., were all
not be more than 3 consecutive school years in the
engaged as Instructor 1. The petitioners executed individual
elementary and secondary levels.
Teacher’s Contracts for each of the trimesters that they were
engaged to teach.
There should be no question that respondent’s employment
was probationary in character consistent with standard
For the school year 2000-2001, AMACC implemented new
practice in private schools. The court cannot subscribe to the
faculty screening guidelines which were used to determine
proposition that she had acquired regular tenure as teacher
the present faculty members’ entitlement to salary increases.
since she only rendered service from April 18, 2002 to March
The petitioners failed to obtain a passing rating based on the
31, 2003 she has not completed the 3-year probationary
performance standards; hence AMACC did not give them
period. Also, her appointment as acting principal was merely
any salary increase. Because of AMACC’s action on the
temporary and hence revocable at will, the employee cannot
salary increases, the petitioners filed a complaint with the
attain security of tenure with respect to such position.
NLRC for underpayment of wages, non-payment of overtime
and overload compensation, 13th month pay, and for
Her resignation also was not valid. Resignation is the
discriminatory practices.
voluntary act of an employee who finds himself in a situation
where he believes that personal reasons cannot be sacrificed
On September 7, 2000, the petitioners individually received
in favor of the exigency of the service, and that he has no
a memorandum from AMACC informing them that with the
other choice but to dissociate himself from employment.
expiration of their contract to teach, their contract would no
There was no showing of an express acceptance of the
longer be renewed. Thus, the petitioners’ labor arbitration
employer, which is what renders resignation operative.
complaint was amended to include the charge of illegal
Furthermore, resignation is inconsistent with the filing of a
dismissal. Petitioners claimed that their dismissal was illegal
complaint for illegal dismissal.
WANGBU2018| BASELERES.CENIZA.CORRO.GAITERA.GARA.MAXINO.VIDAL|Page 17 of 28
because it was made in retaliation for their complaint for renewal (or effectively, the termination of employment of
monetary benefits and discriminatory practices against employees on probationary status) lacks the supporting
AMACC. The petitioners also contended that AMACC finding of just cause that the law requires and, hence, is
failed to give them adequate notice. illegal.

AMACC contended in response that the petitioners worked NOTE (Fixed-term vs. probationary status): The fixed-term
under a contracted term under a non-tenured appointment character of employment essentially refers to the period
and were still within the three-year probationary period for agreed upon between the employer and the employee;
teachers. Their contracts were not renewed for the following employment exists only for the duration of the term and ends
term because they failed to pass the Performance Appraisal on its own when the term expires. In a sense, employment on
System for Teachers (PAST) while others failed to comply probationary status also refers to a period because of the
with the other requirements for regularization, promotion, or technical meaning “probation” carries in Philippine labor
increase in salary. This move, according to AMACC, was law – a maximum period of six months, or in the academe, a
justified since the school has to maintain its high academic period of three years for those engaged in teaching jobs.
standards.
#46 ST. PAUL COLLEGE QUEZON CITY,
The Labor Arbiter ruled that Article 281 of the Labor Code SR. LILIA THERESE TOLENTINO, SPC, SR.
on probationary employment applied to the case and that the
petitioners were illegally dismissed. The NLRC affirmed in BERNADETTE RACADIO, SPC, AND SR.
toto the LA’s ruling, observing however that the applicable SARAH MANAPOL VS. REMIGIO
law is Sec. 92 of the Manual of Regulations for Private MICHAEL A. ANCHETA II AND CYNTHIA
Schools (which mandates a probationary period of nine
consecutive trimesters of satisfactory service for academic
A. ANCHETA
personnel in the tertiary level where collegiate courses are G.R. NO. 16990
offered on a trimester basis), not Article 281 (which SEPTEMBER 7, 2011
prescribes a probationary period of six months). On appeal,
the Court of Appeals disagreed with the LA’s and NLRC’s PERALTA, J.
rulings and held that petitioners were not actually dismissed
but their contracts merely expired and were no longer FACTS: Petitioner St. Paul College, Quezon City (SPCQC),
renewed by AMACC because they failed to satisfy the represented by Sr. Lilia Therese Tolentino, SPC, the College
school’s standards. Dean, Sr. Bernadette Racadio, SPC, and the Mass
Communication Program Director, Sr. Sarah Manapol, SPC,
ISSUE 1: Should the teachers’ probationary status be is a private Catholic educational institution who hired
disregarded simply because the contracts were fixed-term? respondent Remigio Michael (Remigio) as a teacher in the
General Education Department with a probationary rank in
RULING: No. in a situation where the probationary status the School Year (SY) 1996-1997 which was renewed in the
overlaps with a fixed-term contract not specifically used for following SY 1997-1998. His wife, respondent Cynthia was
the fixed term it offers, Article 281 should assume primacy hired by the same school as a part time teacher of the Mass
and the fixed-period character of the contract must give way. Communication Department in the second semester of SY
This conclusion is immeasurably strengthened by the 1996-1997 and her appointment was renewed for SY 1997-
petitioners’ and the AMACC’s hardly concealed expectation 1998.
that the employment on probation could lead to permanent
status, and that the contracts are renewable unless the On February 13, 1998, Remigio and his wife signified his
petitioners fail to pass the school’s standards. intent to renew his contract with SPCQC thru a letter
addressed to Sr. Lilia to which Sr. Bernadette replied, thru a
ISSUE 2: Was the termination of the petitioner done with letter, informing the spouses of the extension of their
just cause? contracts for SY 1998-1999.

RULING: No. While the standards were duly On April 22, 1998, a letter was sent to Sr. Bernadette signed
communicated to the petitioners and could be applied some of the teachers of SPCQC including Remigio and his
beginning the 1st trimester of the school year 2000-2001, wife containing their sentiments regarding two school
glaring and very basic gaps in the school’s evidence still policies. Firstpolicy being, the penalizing the delay in
exist. The exact terms of the standards were never introduced encoding the final grades, and the other being the policy of
as evidence; neither does the evidence show how these withholding salaries of the teachers.
standards were applied to the petitioners. Inevitably, the non-
WANGBU2018| BASELERES.CENIZA.CORRO.GAITERA.GARA.MAXINO.VIDAL|Page 18 of 28
In a letter, contested by Remigio to be ante-dated, dated April Section 92. Probationary Period-- Subject in all
21, 1998 written by Sr. Bernadette to Remigio reiterating the instances to compliance with the Department and school
conversation that took place between them on April 20, 1998. requirements, the probationary period for academic
It enumerated the departmental and instructional policies that personnel shall not be more than Three (3) consecutive years
Remigio failed to comply with, such as the late submission of satisfactory service for those in the elementary and
of final grades, failure to submit final test questions to the secondary levels, six (6) consecutive regular semesters of
Program Coordinator, the giving of tests in the essay form satisfactory service for those in the tertiary level, and nine
instead of the multiple choice format as mandated by the (9) consecutive trimesters of satisfactory service for those in
school and the high number of students with failing grades in the tertiary level where collegiate courses are offered on a
the classes that he handled. trimester basis.

Subsequently, Sr. Bernadette wrote a letter dated April 30, It must be noted that contract of probationary employment
1998 to Sr. Lilia endorsing the immediate termination of the must specify the period or term of its effectivity, failure to
teaching services of Remigio and his wife on the grounds stipulate its precise duration could mean that the contract is
cited by Sr. Bernadette in her letter to Remigio to which the binding for the full three year probationary period.
latter, together with his wife, were given the opportunity to Therefore, the letters sent by SPCQC, which were void of
comment. Spouses sent their respective comments to Sr. any specifics, cannot be considered as contracts. As such,
Lilia and subsequently received their letters of termination. petitioner school has the right not to renew the contracts of
They sent for a letter of reconsideration which was denied. the spouses, the old ones having been expired at the end of
their terms.
Spouses filed for illegal dismissal with the NLRC against
SPCQC, to which the labor arbiter dismissed for lack of NOTE: The common practice is for the employer and the
merit. teacher to enter into a contract, effective for one school year.
At the end, the employer has the option not to renew the
CA granted the petition and reversed the decision of the labor contract, particularly considering the teacher’s
arbiter due to finding of grave abuse of discretion, ordering performance. If the contract is not renewed, the employment
SPCQC to pay separation pay, deficiency wages, and other relationship terminates. If the contract is renewed, usually
damages. for another school year, the probationary employment
continues. Again, at the end, the parties may renew or not
SPCQC contends that it did not extend the contracts of the contract. If renewed, this second renewal of the contract
respondent spouses. It claims that, although, it has sent letters for another school year would then be the last year since it
to the spouses informing them that the school is extending to would be the third school year of probationary employment.
them new contracts for the coming school year, the letters do At the end of this third year, the employer may now decide
not constitute as actual employment contracts but merely whether to extend a permanent appointment to the employee,
offers to teach on the said school year. primarily on the basis of the employee having met the
reasonable standards of competence and efficiency set by the
ISSUE: Was there illegal dismissal? employer. For the entire duration of this three year period,
the teacher remains under probation. Upon the expiration of
RULING: No. Remegio was a full-time probationary his contract of employment, being simply on probation, he
teacher, while his wife was a part-time teacher. The cannot automatically claim security of tenure and compel the
probationary employment is intended to afford the employer employer to renew his employment contract.
an opportunity to observe the fitness of a probationary
employee while at work, and to ascertain whether he will Assuming however that there was indeed employment
become an efficient and productive employee. Thus, the contract, there was still valid and just cause for the dismissal.
word probationary, as used to describe the period of The Labor Code provides that an employer may legally
employment, implies the purpose of the term or period, not dismiss an employee when the requirement of substantial and
its length. procedural due process are complied with. In this case,
SPCQC charged respondent Remigio of non-compliance
Employment on probationary status of teaching personnel with a school policy regarding the submission of final test
are not governed purely by the Labor Code. The Labor Code questions to his program coordinator in writing for checking
is supplemented with respect to the period of probation by or comment to which the latter admitted.
special rules found in the Manual of Regulations for Private
Schools. Such admissions were considered by the SPCQC in not
renewing the contracts. It is the prerogative, tone that must
Under Section 92 of said manual provides: be exercised in good faith, of the school to set high standards
WANGBU2018| BASELERES.CENIZA.CORRO.GAITERA.GARA.MAXINO.VIDAL|Page 19 of 28
of efficiency for its teachers since quality education is a paper with the words “RSD” as the cause of his termination,
mandate of the Constitution as long as the standards fixed are and he did not understand the cause nor was it explained to
reasonable and not arbitrary, courts are not at liberty to set him.
them aside. Schools cannot be required to adopt standards
which barely satisfy criteria set for government recognition. Petitioner, D.M. Consunji Corporatation (DMCI), Defenses:
The same academic freedom grants the school the autonomy Petitioner, DMCI contends that Rogelio was not a regular
to decide for itself the terms and conditions for hiring its employee but had only been a project employee.
teacher, subject of course to the overarching limitations Petitioner, DMCI contends that he had tendered his voluntary
under the Labor Code. The authority to hire is likewise resignation for health reasons which rendered him incapable
covered and protected by its management prerogative the of performing his job according to the resignation letter.
right of an employer to regulate all aspects of employment,
such as hiring, the freedom to prescribe work assignments, DMCI contends that there were considerable and substantial
working methods, process to be followed, regulation gaps between complainant’s employments. And that it was
regarding transfer of employees, supervision of their work, not improbable that the complainant may be hired
lay-off and discipline, and dismissal and recall of workers. continuously one after the other because DMCI is one of the
CENIZA biggest and well known construction companies.
DMCI also contends that the project employment contract
they then entered into clearly gave notice to him at the time
#47 D.M CONSUNJI CORPORATIN VS. of his engagement about his employment being for a specific
BELLO project or phase of work. He was also thereby notified of the
duration of the project, and the determinable completion date
(Extension of employment of a project employee long after
of the project.
the supposed project completion, makes him a regular
employee)
Decisions:
G.R. NO. 159371
ELA (Executive Labor Arbiter)
JULY 29, 2013
Respondent, Rogelio A. Bello, filed with the Executive
Labor Arbiter a case for illegal dismissal and damages. The
BERSAMIN, J.
ELA ruled in favour of Respondent, Rogelio A. Bello,
declaring DMCI guilty of illegal dismissal.
FACTS: Respondent, Rogelio P. Bello was a mason of D.M
Consunji Corporation (DMCI), a construction company.
NLRC
Petitioner, DMCI appealed to the NLRC. The NLRC ruled
Respondent, Rogelio P. Bello, was successively hired for
in favour of DMCI and dismissed Respondent, Rogelio A.
various construction projects of DMCI, one after the other.
Bello’s claims. The NLRC ruled that Rogelio was a project
He was diagnosed to be suffering from pulmonary
employee. The NLRC found that there were considerable and
tuberculosis thereby necessitating his leave of absence. Upon
substantial gaps between complainant’s employment.
his recovery, he reported back to work, but DMCI had
refused to accept him and had instead handed to him a
Court of Appeals
termination paper.
Respondent, Bello, petitioned to the CA, CA found that Bello
acquired the status of a regular employee. His repeated re-
Hence, Repondent, Rogelio P. Bello filed a case for illegal
hiring and the continuing need for his services over a long
dismissal and damages against DMCi and/or Rachel
span of time had made him a regular employee.
Consunji.
Hence DMCI appeals to the SC
Assertions and Defenses:
Respondent, Rogelio Bello, Assertions:
ISSUE: Whether or not Rogelio Bello is a regular employee
Rogelio contends that he had not been given prior notice of
or a project employee
his termination and did not know the cause of his
termination.
RULING: Yes,respondent, Rogelio is a regular employee.
Respondent, Rogelio contends that he was hired as a mason
Rogelio was initially a project employee of DMCI but
without any interruption from February 1, 1990 until October
acquired in time the status of a regular employee by virtue of
10, 1997. He asserts that his job had been necessary and
his continuous work as a mason of DMCI. His work was
desirable in the usual business or trade of DMCI.
found to be a function necessary and desirable to the business
Respondent, Rogelio contends that DMCI had refused to
or trade. This is evidence by DMCI hiring him in not just
accept him and had instead handed to him a termination
one, but several of its projects for a span of about 8 years.
WANGBU2018| BASELERES.CENIZA.CORRO.GAITERA.GARA.MAXINO.VIDAL|Page 20 of 28
to prepare an incident report to explain the reason for the said
The length of time of the employee’s service, while not a oversight. In the afternoon of that same day, private
controlling determinant of project employment, is a strong respondent was summoned to get his separation pay of
factor in determining whether he was hired for a specific P55,000.00. However, he refused to accept the amount as he
undertaking or in fact tasked to perform functions vital, believed that he did nothing illegal to warrant his immediate
necessary and indispensable to the usual business or trade of discharge from work.
the employer.
On July 11, 2000, private respondent filed a complaint for
A project employee is, one who is here for a specific project illegal dismissal with the Labor Arbiter. Petitioners however
or undertaking and the completion or termination of such asserted that private respondent was a contractual or a casual
project or undertaking has been determined at the time of employee whose services could be terminated at the end of
engagement of the employee. the contract even without a just or authorized cause.
Asserting their right to terminate the contract with private
The principal test for determining whether an employee is a respondent per the “Kasunduan” with him, petitioners
project employee, as distinguished from a regular employee, pointed to the provision thereof stating that he was being
is whether or not he is assigned to carry out a specific project employed only on a “por viaje” basis and that his
or undertaking, employment would be terminated at the end of the trip for
The duration and scope of which are specified at the time he which he was being hired.
is engaged for the project. In regards to this principle test, the
court did not treat Rogelio’s repeated engagements The Labor Arbiter held that even if the private respondent
separately but took into consideration that his successive was a casual employee, he became a regular employee after
reengagement in order to perform the same kind of work as a period of one year and, thereafter, had attained tenurial
a mason firmly manifested the necessity and desirability of security which could only be lost due to a legal cause after
his work in DMCI’s usual business of construction. observing due process. The LA’s Decision was also affirmed
by the NLRC and the Court of Appeals.
Petitioner, DMCI by failing to prove the alleged voluntary
resignation of the employee in the illegal dismissal case by ISSUE: Was private respondent a regular employee at the
clear, positive and convincing evidence renders the case time his employment was terminated?
infavor of respondent, Rogelio.
RULING: Yes. In a span of 12 years, private respondent
#48 POSEIDON FISHING VS. NATIONAL worked for petitioner company first as a Chief Mate, then
LABOR RELATIONS COMMISSION Boat Captain, and later as Radio Operator. His job was
directly related to the deep-sea fishing business of petitioner
G.R. NO. 168052
Poseidon. His work was, therefore, necessary and important
FEBRUARY 20, 2006
to the business of his employer. Such being the scenario
involved, private respondent is considered a regular
CHICO-NAZARIO, J.
employee of petitioner under Article 280 of the Labor Code.
FACTS: Private respondent Estoquia was employed by
At different times, private respondent occupied three
Poseidon Fishing in January 1988 as Chief Mate. After five
different positions. Such pattern of re-hiring and the
years, he was promoted to Boat Captain, but in 1999, he was
recurring need for his services are testament to the necessity
demoted to Radio Operator. As a Radio Operator, he
and indispensability of such services to petitioners’ business
monitored the daily activities in their office and recorded in
or trade.
the duty logbook the names of the callers and time of their
calls.
NOTE: The acid test in considering fixed-term contracts as
valid is: if from the circumstances it is apparent that periods
On 3 July 2000, private respondent failed to record a 7:25
have been imposed to preclude acquisition of tenurial
a.m. call in one of the logbooks. However, he was able to
security by the employee, they should be disregarded for
record the same in the other logbook. Consequently, when he
being contrary to public policy. (Brent School, Inc. vs.
reviewed the two logbooks, he noticed that he was not able
Zamora)
to record the said call in one of the logbooks so he
immediately recorded the 7:25 a.m. call after the 7:30 a.m.
entry. Around 9:00 o’clock in the morning of 4 July 2000, #49 OMNI HAULING SERVICES VS
petitioner de Jesus (Manager) detected the error in the entry BERNARDO BON
in the logbook. Subsequently, she asked private respondent G.R. NO. 199388
WANGBU2018| BASELERES.CENIZA.CORRO.GAITERA.GARA.MAXINO.VIDAL|Page 21 of 28
SEPTEMBER 3, 2014 (2) a particular job or undertaking that is not within the
regular business of the corporation. In order to safeguard the
PERLAS-BERNABE, J: rights of workers against the arbitrary use of the word
„project‰ to prevent employees from attaining a regular
FACTS: Petitioner, Omni Hauling Services Inc was status, employers claiming that their workers are project
awarded a one year service contract by the local government employees should not only prove that the duration and scope
of Quezon to provide garbage hauling service from July 2002 of the employment was specified at the time they were
to June 2003. For this purpose, petitioner hired respondents engaged, but also that there was indeed a project
as garbage truck drivers and paleros who were paid on a per
trip basis. When the service contract was renewed, the ISSUE 2: Are respondents regular employees?
respondents were required to sign employment contracts
which provided that they will be rehired only for the duration RULING 2: YES. In this case, records are bereft of any
of the same period. Respondents refused to sign claiming that evidence to show that respondents were made to sign
they were regular employees since they were engaged to employment contracts explicitly stating that they were going
perform activities which were necessary and desirable to to be hired as project employees, with the period of their
Omni’s usual business or trade. Thus, petitioner terminated employment to be coterminous with the original period of
the employment of respondents. OmniÊs service contract
with the Quezon City government. Neither is petitionersÊ
LABOR ARBITER: Ruled that respondents were not allegation that respondents were duly apprised of the project-
illegally dismissed. The LA found that respondents, at the based nature of their employment supported by any other
time of their engagement, were informed that their evidentiary proof. Thus, the logical conclusion is that
employment will be limited for a specific period of one year respondents were not clearly and knowingly informed of
and was coterminous with the service contract with the their employment status as mere project employees, with the
Quezon City government. duration and scope of the project specified at the time they
were engaged.
NLRC: Sustained the LA’s findings. Thus, when
respondents refused to sign the employment contracts for the As such, the presumption of regular employment should be
subsequent period, there was no dismissal to speak of, but accorded in their favor pursuant to
rather, a mere expiration of respondents’ previous contracts. Article 280 of the Labor Code which provides that
Thus, respondents were not regular but merely project “[employees] who have rendered at least one year of
employees whose hiring was solely dependent on the service, whether such service is continuous or broken [· as
aforesaid service contract. respondents in this case ·] shall be considered as [regular
employees] with respect to the activity in which [they] are
CA: Reversed NLRC’s pronouncements. It held that the employed and [their] employment shall continue while such
NLRC failed to consider the glaring fact activity actually exists.” Add to this the obvious fact that
that no contract of employment exists to support petitioners’ respondents have been engaged to perform activities which
allegation that respondents are fixed-term (orproperly are usually necessary or desirable in the usual business or
speaking, project) employees. trade of Omni, i.e., garbage hauling, thereby confirming the
strength of the aforesaid conclusion.
ISSUE 1: What is the test of determining project employees?
The determination that respondents are regular and not
RULING 1: According to jurisprudence, the principal test merely project employees resultantly means that theiservices
for determining whether particular employees are properly could not have been validly terminated at the expiration of
characterized as “project employees” as distinguished from the project, or, in this case, the service contract of Omni with
regular employees is whether or not the employees were the Quezon City government. Petitioners in failing to prove
assigned to carry out a specific project or undertaking, the that have been dismissed for just cause warrants the
duration (and scope) of which were specified at the time they conclusion that respondents were illegally dismissed.
were engaged for that project. The project could either be:
#51 PNOC-ENERGY DEV. CORP. VS NLRC
(1) a particular job or undertaking that is within the regular (The true test in determining project employment)
or usual business of the employer company, but which is G.R. NO. 169353
distinct and separate, and identifiable as such, from the other APRIL 13, 2007
undertakings of the company; or
CALLEJO, SR., J.

WANGBU2018| BASELERES.CENIZA.CORRO.GAITERA.GARA.MAXINO.VIDAL|Page 22 of 28
FACTS: Petitioner PNOC Energy Development In this case, the specific project or undertaking of petitioner
Corporation (PNOC) is a GOCC engaged in the exploration, was not satisfactorily identified (as it appears in the contract,
development, and utilization of energy in particular it lists project names as: Maintenance of drilling materials,
geothermal energy. Each geothermal project undergoes EDC drilling activites, etc..) The court finds the descriptions
stages of exploration, development, utilization, or production as too vague or imprecise to be considered as “specific
and for each stage activities such as drilling, construction, undertaking”. The act of repeatedly and continuously hiring
civil works, structural works, etc. are undertaken until the respondents to do the same kind of work also belies their
project is completed. Petitioner undertakes projects in Neg. contention that respondents were hired for a specific project
Or., Neg. Occ., Leyte, Albay, etc. For instance, petitioner’s or undertaking. In addition, the contracts were extended a
southern negros geothermal production filed in Neg. Or. is number of times for different or new projects also. Thus, it is
divided into 2 phases namely: Palinpinon I and II (PAL I and not the length of service of a project employee which
II). Petitioner hired respondents to augment the manpower constitutes as the controlling test of employment tenure but
requirement in the development of PAL II, namely: Leonora it is “whether or not the employment has been fixed for a
Torres, Rosela Calimpong, Arnel Amor, Wilson Nuay, specific project or undertaking the completion or termination
Roberto Renzal, and Alejandro Tabañera. The termination of of which has been determined at the time of the engagement
their respective employment were specified in their initial of the employee”.
contracts which were renewed and extended on their expiry.
On May 29, 1998 petitioner submitted to DOLE that ISSUE #2: Were they illegally dismissed?
respondents were being terminated for substantial
completion of the civil works phase of PAL II. The RULING: Yes. As regular employees, respondents are
respondents however filed a complaint for illegal dismissal. entitled to security of tenure under Art. 279 of the Labor
Code and can only be dismissed for a just or authorized
Respondents averred that they had rendered continuous and cause. In termination cases, it is the employer who has the
satisfactory services from the dates of their respective burden of proving that the dismissal was not illegal and in
employment until illegally dismissed on June 30, 1998 (A the case at bar they failed to discharge such burden. It appears
table shows that at most they have rendered 2 years and 4 that the notices of termination indicated that respondents
months of service with some reaching 3 years and 5 months). were terminated due to the completion of the project however
They also averred that this is a clear case of union busting this is contrary to the statement of petitioner that the project
since they previously sought union membership and even was merely “substantially completed”. /DIORSON
filed a notice of strike. On the other hand, petitioner avers
that respondents were project employees because they were #52 ALCATEL PHILIPPINES, INC. VS.
hired for a specific project or undertaking, the completion or RELOS
termination of which had been determined at the time of their
G.R. NO. 164315
engagement.
JULY 3, 2009
The Labor Arbiter dismissed the complaint for lack of legal
CARPIO, J.
and factual basis finding that the relationship was severed
upon expiration of the respective contracts and completion
FACTS: Relos (respondent) was offered temporary
of the projects concerned. But the NLRC and CA rendered a
employment by Alcatel (petitioner) initially as
decision in favor of respondents, stating that they were
Estimator/Draftsman to assist in the preparation of manholes
regular nonproject employees for working more than 1 year
and conduit design for the proposal preparation for PLDT X-
in positions that required them to perform activities
5 project for the period of January-February 1988. Such
necessary and desirable in the normal business or trade of
temporary employment was again offered for the period of
petitioner. It is according to the NLRC an illegal dismissal
March to April 1988. Subsequently, petitioner undertook the
made under the pretext of project completion.
PLDT 1342 project which involved the installation of
microwave antennas and towers in Eastern Visayas and
ISSUE: Were private respondents project employees or
Eastern Mindanao. For such purpose, respondent was offered
regular employees?
temporary employment as Civil Works Inspector (February
to March 1991), as Civil Works Engineer (April 1991 to July
RULING: Regular Employees. As defined, project
1992), and as Civil Engineer (August 1992 to December
employees are those workers hired (1) for a specific project
1993). Due to delays of the project, petitioner extended
or undertaking, and (2) the completion or termination of such
respondent’s employment for another three months until
project or undertaking has been determined at the time of the
March 1994. Thereafter, petitioner employed respondent as
engagement of the employee.
WANGBU2018| BASELERES.CENIZA.CORRO.GAITERA.GARA.MAXINO.VIDAL|Page 23 of 28
a Site Inspector until December 1995. In December 1995,
petitioner informed respondent that the project would then be ISSUE: Was respondent a regular employee or a project
completed and that his contract would expire. Petitioner employee?
asked respondent to settle all his accountabilities with the
company and advised him that he would be called if it has RULING: Respondent was a project employee. The specific
future projects that require his expertise. projects for which respondent was hired and the periods of
employment were specified in his employment contracts.
In March 1997, respondent filed a complaint for illegal The services he rendered, the duration and scope of each
dismissal, separation pay, unpaid wages, unpaid overtime employment are clear indications that respondent was hired
pay, damages, and attorney’s fees against petitioner. as a project employee.
Respondent alleged that he was a regular employee of
Alcatel and that he was dismissed during the existence of the The principal test for determining whether a particular
project. employee is a project employee or a regular employee is
whether the project employee was assigned to carry out a
Petitioner argues that respondent was a project employee specific project or undertaking, the duration and scope of
because he worked on distinct projects with the terms of which were specified at the time the employee is engaged for
engagement and the specific project made known to him at the project. “Project” may refer to a particular job or
the time of the engagement. Alcatel clarifies that undertaking that is within the regular or usual business of the
respondent’s employment was coterminous with the project employer, but which is distinct and separate and identifiable
for which he was hired and, therefore, respondent was not as such from the undertakings of the company. Such job or
illegally dismissed but was validly dismissed upon the undertaking begins and ends at determined or determinable
expiration of the term of his project employment. Alcatel times.
explains that its business relies mainly on the projects it
enters into and thus, it is constrained to hire project In Maraguinot, Jr. v. NLRC, the Court said:
employees to meet the demands of specific projects.
A project employee or a member of a work pool may acquire
LABOR ARBITER – Since respondent was repeatedly hired the status of a regular employee when the following concur:
by Alcatel, respondent performed functions that were 1) There is a continuous rehiring of project employees even
necessary and desirable in the usual business or trade of after the cessation of a project; and
petitioner. The Labor Arbiter concluded that respondent 2) The tasks performed by the alleged “project employee”
belonged to the “work pool of non-project employees” of are vital, necessary and indispensable to the usual business
petitioner. or trade of the employer.

NLRC – The NLRC set aside the Labor Arbiter’s ruling and While respondent performed tasks that were clearly vital,
declared that respondent was a project employee. The NLRC necessary and indispensable to the usual business or trade of
said respondent was assigned to carry out a specific project Alcatel, respondent was not continuously rehired by Alcatel
or undertaking and the duration of his services was always after the cessation of every project. Alcatel’s continuous
stated in his employment contracts. The NLRC also pointed rehiring of respondent in various capacities from February
out that, by the nature of petitioner’s business, respondent 1991 to December 1995 was done entirely within the
would remain a project employee regardless of the number framework of one and the same project―the PLDT 1342
of projects for which he had been employed. Since project.
respondent was a project employee, the NLRC said he was
not illegally dismissed, but that his dismissal was brought ISSUE 2: Was respondent illegally dismissed?
about by the expiration of his employment contract.
RULING: No. The employment of a project employee ends
COURT OF APPEALS – CA set aside the NLRC’s decision on the date specified in the employment contract. Therefore,
and reinstated the Labor Arbiter’s ruling. Respondent was a respondent was not illegally dismissed but his employment
regular employee of petitioner because (1) he was assigned terminated upon the expiration of his employment contract.
to positions and performed tasks that were necessary to the /GARA
main line and business operations of petitioner; (2) he was
repeatedly hired and contracted, continuously and for
prolonged periods, with his employment contracts renewed
each time they fell due; and (3) petitioner did not report the
termination of the projects with the nearest public
employment office.
WANGBU2018| BASELERES.CENIZA.CORRO.GAITERA.GARA.MAXINO.VIDAL|Page 24 of 28
#53 HERMA SHIPYARD, INC. AND MR. the project for which they were hired and not for a uniform
period of one month.
HERMINIO ESGUERRA VS DANILO
OLIVEROS, JOJIT BESA, ARNEL SABAL, CA granted the stating that the respondents have become
CAMILO OLIVEROS, ROBERT NARIO, regular employees since they were performing tasks that are
FREDERJCK CATIG, RICARDO ONTALAN, necessary desirable, and vital to the operation of HS’
business. The latter also failed to present proof that
RUDEN DELGADO, SEGUNDO LABOSTA, respondents were hired for a specific period or that their
EXEQUIEL OLIVERIA OSCAR TIROL AND employment was coterminous with a specific project, such
ROMEO TRINIDAD was not clear from the contracts presented that the
G.R. NO. 208936 completion or termination of the project or undertaking was
APRIL 17, 2017 already determined at the time petitioners engaged the
services of respondents; the latter were made to work not
DEL CASTILLO, J: only in one project but also in different projects and were
assigned to different departments of HS; respondents
FACTS: Herma Shipyard (HS), petitioner, is a domestic repeatedly and successively rehired as employees HS; except
corporation engaged in the business of shipbuilding and with regard to respondents' last employment, HS failed to
repair while respondents were it s employees. On June 17, present proof that they reported to the nearest public
2009, the respondents filed before the Regional Arbitration a employment office the termination of respondent’s previous
Complaint for illegal dismissal, regularization, and non- employment or every time a project or a phase thereof had
payment of service incentive leave pay with prayer for the been completed; and, petitioners failed to file as many
payment of full backwages and attorney's fees against HS. reports of termination as there were shipbuilding and repair
Respondents alleged that they are HS’ regular employees projects actually completed. CA concluded that HS did try to
who have been continuously performing tasks usually circumvent the respondent’s right to security of tenure.
necessary and desirable in its business. However, they were
dismissed by HS on various dates. HS contends that necessity and desirability of respondents'
services in HS’ business are not the only factors to be
Respondents also alleged that HS made them sign considered in determining the nature of respondents'
employment contracts for a fixed period ranging from 1 to 4 employment but also the contracts of employment signed by
months to make it appear that they were project-based the respondents apprising them of the fact that their services
employees as a condition to their employment. They said that were engaged for a particular project only and that it was
it was a scheme to defeat their right to security of tenure, coterminous with it. The authenticity and genuineness of the
when in truth, they never stopped working for HS. And contracts were never disputed by the respondents during the
finally, they said that were they actually project based pendency of the case before the labor tribunals.
employees, HS should have reported to the DOLE the
completion of the project but no report was submitted. It also added that the employment contract shows that the
employment was dependent upon the completion of the
As a defense, HS argued that respondents were its project- project indicated. And with regard to the repeated rehiring,
based employees in its shipbuilding projects and that the HS insist it will not result in becoming regular employees
specific project for which they were hired had already been because length of service does not determine employment
completed. HS even presented the contract of employment. status. What is controlling of project-based employment is
whether the employment has been fixed for a specific project
The labor arbiter dismissed the complaint ruling that the or undertaking, its completion having been determined and
respondents were project based whose services were made known to the employees at the time of their
properly terminated upon the completion of the specific work engagement.
for which they were hired. Such decision was affirmed by the
NLRC in toto and the consequent MR denied. ISSUE: What is the nature of respondents’ employment with
HS?
They appealed to the court of appeals citing grave abuse of
discretion, stating the labor tribunals erred in relying on the RULING: Project based employees. A project employee
project employment contracts which were for a uniform under Article 280 (now Article 294) of the Labor Code, as
duration of one month. They argued that if it were true that amended, is one whose employment has been fixed for a
they were project-based employees, the duration of their specific project or undertaking, the completion or
employment should have coincided with the completion of termination of which has been determined at the time of the
engagement of the employee. The services of project based
WANGBU2018| BASELERES.CENIZA.CORRO.GAITERA.GARA.MAXINO.VIDAL|Page 25 of 28
employees are coterminous with the project and may be FACTS: Respondent Pontesor et al. , performs various
terminated upon the end or completion of the project or a maintenance duties within petitioner UST’s campus such as
phase thereof for which they were hired. being a labourer, mason, painter, electrician, carpenter. Etc.
Petitioner, UST is an educational institution with buildings,
The principal test in determining whether particular rooms and facilities being maintained by respondent
employees were engaged as project-based employees, as Pontesor et al.
distinguished from regular employees, is whether they were
assigned to carry out a specific project or undertaking, the Respondent Pontesor and Samahang Manggagawas ng UST
duration and scope of which was specified at, and made filed a complaint for regularization and illegal dismissal
known to them, at the time of their engagement. against UST.

The records of this case reveal that for each and every project Assertions/ Defenses:
respondents were hired, they were adequately informed of Respondent Pontesor Assertions:
their employment status as project based employees at least Respondent Pontesor et al. asserts that they perform
at the time they signed their employment contract. They were maintenance duties in the school campus and the
fully apprised of the nature and scope of their work whenever performance of such duties throughout the years made them
they sign their employment contract. Their contracts (mostly regular employees of petitioners. And such maintenance
written in the vernacular) provide in no ambiguity as to the tasks are necessary and desirable to the business of the
terms but state clearly the date of the commencement of the Petitioner
specific task and the expected completion date. It also
contain a provision expressly stating that respondents' Petitioner UST defences:
employment shall end upon the arrival of the target Petitioner, UST admits that it repeatedly hired Pontesor, et
completion date or upon the completion of such project. al. but nevertheless maintained that they were merely hired
on a per-project basis, as evidence by the numerous
With regard to the issue of performing tasks necessary and Contractual Employee Apointments (CEA).
desirable to the business operation of the employer, it is
settled that project-based employees may or may not be Petitioner, UST asserts that the CEA defined the nature and
performing tasks usually necessary or desirable in the usual term of the project to which they are assigned, each contract
business or trade of the employer. The fact that the job is was renewable if the project remained unfinished upon the
usually necessary or desirable in the business operation of expiration of the specified term.
the employer does not automatically imply regular
employment; neither does it impair the validity of the project Decisions:
employment contract stipulating a fixed duration of Labor Arbiter’s Ruling: Samahang Manggagawas ng UST
employment. and Pontesor filed with the NLRC a complaint for
regularation and illegal dismissal. LA ruled in favor of
As to the issue of rehiring, the rule that employees initially Respondent Pontesor. That Respondent Pontesor should be
hired on a temporary basis may become permanent deemed as petitioner UST’s employee. And that there was
employees by reason of their length of service is not illegal dismissal
applicable to project -based employees. This is because it
would be unjust to require the employer to maintain NLRC’s Ruling: NLRC found that Respondent Pontesor et
employees in their payroll even after the completion of the al. cannot be considered as regular employees as they
project since this would be tantamount to making the knowingly and voluntarily entered into fixed term contracts
employee a privileged retainer who collects payment from of employment with petitioner.Hence there could be no
his employer for work not done, and amounts to labor illegal dismissal. Respondent Pontesor was classified as a
coddling at the expense of management. CENIZA fixed term casual employee.

#54. UNIVERSITY OF SANTO TOMAS VS Court of Appeals: Ruled in favour of Pontesor, set aside the
SAMAHANG MANGGAGAWA NG UST NLRC ruling and reinstated the Labor Arbiter ruling.
Respondent, Pontesor, et al. cannot be considered as merely
(Project Employees, Regularized Casual Employees)
fixed term or project employees since they performed work
G.R. NO. 184262
that is necessary and desirable and the the project description
APRIL 24, 2017
set forth in their respective CEA’s were either too general or
too broad.
Perlas-Bernabe, J;

WANGBU2018| BASELERES.CENIZA.CORRO.GAITERA.GARA.MAXINO.VIDAL|Page 26 of 28
ISSUES: Whether or not Respondent Pontesor, et al. are Respondent , Servana filed a complaint for illegal dismissal
regular employees and, consequently were illegally against TAPE.
dismissed by petitioner UST.
Assertions and Defenses:
RULING: Yes, Respondent Pontesor et. al are regularized Assertions of Respondet Servana:
casual employees. There are two categories of regular Respondent, Servana, a security guard alleges that he was
employees, (1) those who perform usually necessary or first connected with Agro-Commercial Security Agency but
desirable activities in the usal business or trade of the was later on absorbed by TAPE as a regular company guard.
employer and (2) those who have rendered at least 1 year of He asserts that he was a regular employee having been
service, whether continuous or broken, with respect to the engaged to perform an activity that is necessary and desirable
activity in which they were employed. to TAPE’s business for thirteen years.

Respondent Pontesor’s nature of work are not necessary and Defenses of Petitioner TAPE:
desirable to petitioner’s usual businesses as an educational Petitioner TAPE,contends that there was no employer-
institution , however, It is clear that their respective employee relationship between parties.
cumulative periods of employment as per their respective Petitioner TAPE, averred that Respondent Servana was an
CEAs each exceed one (1) year. independent contractor.
Petitioner TAPE, asserts that it was Respondent Servana who
Repondent Pontesor cannot also be deemed as project offered his services as a talent to TAPE
employees because the specific undertakings or projects for Petitioner TAPE, advances that the Memorandm served on
which they were employed were not clearly delineated. The Respondent Servana was for discontinuance of the contract
principal test for determining project based employees from for security and not a termination letter.
regular employees is whether or not the employees were Petitioner TAPE insists that they had no control over the
assigned to carry out a specific project or undertaking, the means and methods by which respondent Servana manages
duration (and scope) of which were specified at the time they the live audiences and the safety of its guests.
were engaged for that project.
Petitioner TAPE forwards the claim that Respondent,
The project could either be Servana was employed as a security guard for RPN-9 and
(1) A particular job or undertaking that is within the regular was tasked to assist TAPE. when RPN-9 severed its
or usual business of the employer company, but which is relationship with the security agency, TAPE engaged
distinct and separate, and identifiable as such, from the other Respondent Servana to provide security service to its staff,
undertakings of the company stars and guests until TAPE was able to engage the services
(2) A particular job or undertaking that is not within the of a professional agency, as agreed.
regular business of the corporation.
Labor Arbiter: Labor Arbiter ruled in favour of Servana.
The employers must prove that the duration and scope of the Respondent Servana was a regular employee of TAPE
employment was specified at the time they were engaged and performing necessary and desirable functions in the usual
that there was indeed a project. business activity of TAPE. The Termination was valid on the
ground of redundancy of functions.
Petitioner UST failed to dispense of this requirement because
of the vagueness of their project descriptions NLRC: NLRC ruled against Servana. The NLRC finds that
Respondent Servana was a mere program employee. There
#55 TELEVISION AND PRODUCTION was no reasonable connection between his security services
EXPONENTS, INC. VS. SERVANA and the production of television shows. He did not observe
working hours and worked for other companies.
FACTS: Petitioner, Television and Products Exponents
Court of Appeals: Court of appeals ruled in facor of Servana.
(TAPE) is a domestic corporation, engaged in the production
The court of appeals found respondent to be a regular
of tv programs Respondent, Servana had served as a security
employee.
guard for tape for 13 years, From March 1987 to March 3,
2000. He was dismissed on March 3, 2000 when he received
ISSUE: Whether an employer-emloyee relationshop exists
a memorandum informing him of his impeding dismissal on
between TAPE and respondent Servana?
account of TAPE’s decision to contract the services of a
professional security agency.
Yes, Respondent Servana is a regular employee, The court
used the four-fold test.
WANGBU2018| BASELERES.CENIZA.CORRO.GAITERA.GARA.MAXINO.VIDAL|Page 27 of 28
1) Selection and hiring:
Petitioner TAPE, themselves, admitted to having engaged
the services of respondent servana and acknowledged
respondent servana to be their employee by issuing a
memorandum terminating his services

2) Payment of Wages
Respondent, Servana’s wages were given to him monthly.

3) Control
The time cards of respondent, Servana showed that Petitioner
TAPE had control of the time when Respondent, Servana
should work.

Respondent Servana could also not be an independent


contractor because he did not possess substantial capital or
investment, not present a written contract.

Regardless if respondent, Servana is not performing work


that is necessary or desirable to the usual business of TAPE,
respondent is still considered as regular employee under
Article 280 of the Labor Code, such that any employee who
has rendered at least one year of service, whether such
service is continuous or broken, shall be considered a regular
employee.

As a regular employee, respondent Servana cannot be


terminated except for just cause. His termination was based
on redundancy. The law requires, under art. 283 that a
service of written notice to the DOLE at least one month
prior to the intended date of retrenchment. Petitioner, TAPE
failed to show that it dispensed with this requirement. Hence
there was a procedural infirmity, this does not invalidate the
dismissal but the employer is held loable for the
noncompliance with the procedural requirements.

WANGBU2018| BASELERES.CENIZA.CORRO.GAITERA.GARA.MAXINO.VIDAL|Page 28 of 28

You might also like